Download as pdf or txt
Download as pdf or txt
You are on page 1of 234

A Case-Based Approach

RESYNCHRONIZATION
PACEMAKERS, ICDs, AND CARDIAC
,

TO Pacemakers, ICDs, AND

Cardiac Resynchronization Volume 1

Questions for Examination Review and Clinical Practice

Edited by
Paul A. Friedman MD, FACC, FHRS ­| Melissa A. Rott RN
Anita Wokhlu MD | Samuel J. Asirvatham MD, FACC, FHRS
David L. Hayes MD, FACC, FHRS
A Case-Based Approach to Pacemakers, ICDs, and Cardiac Resynchronization
A Case-Based Approach
TO Pacemakers, ICDs, AND

Cardiac Resynchronization Volume 1

Questions for Examination Review and Clinical Practice

Edited by
Paul A. Friedman MD, FACC, FHRS ­| Melissa A. Rott RN
Anita Wokhlu MD | Samuel J. Asirvatham MD, FACC, FHRS
David L. Hayes MD, FACC, FHRS
© 2011 Mayo Foundation for Medical Education and Research
First paperback edition, 2013

Cardiotext Publishing, LLC


3405 W. 44th Street
Minneapolis, Minnesota 55410 USA

www.cardiotextpublishing.com

Any updates to this book may be found at: www.cardiotextpublishing ments and are urged to review the latest instructions and warnings for
.com/titles/detail/9781935395812. any medicine, equipment or medical device. Readers should consult
with a specialist or contact the vendor of any medicine or medical device
Comments, inquiries, and requests for bulk sales can be directed to the where appropriate.
publisher at: info@cardiotextpublishing.com.
Except for the publisher’s website associated with this work, the publish-
All rights reserved. No part of this book may be reproduced in any form er is not affiliated with and does not sponsor or endorse any websites,
or by any means without the prior permission of Mayo Foundation for organizations or other sources of information referred to herein.
Medical Education and Research. Direct permissions questions to Scien-
tific Publications, Plummer 10, Mayo Clinic, 200 First Street SW, Roch- The publisher and the author specifically disclaim any damage, liability,
ester, MN 55905. or loss incurred, directly or indirectly, from the use or application of any
of the contents of this book.
All trademarks, service marks, and trade names used herein are the
property of their respective owners and are used only to identify the Unless otherwise stated, all figures and tables in this book are used cour-
products or services of those owners. tesy of the authors.

This book is intended for educational purposes and to further general Cover and book design: Ann Delgehausen, Trio Bookworks
scientific and medical knowledge, research, and understanding of the
conditions and associated treatments discussed herein. This book is not Library of Congress Control Number: 2011925451
intended to serve as and should not be relied upon as recommending or ISBN: 978-1-935395-81-2
promoting any specific diagnosis or method of treatment for a particular
condition or a particular patient. It is the reader’s responsibility to deter- Printed in the United States of America
mine the proper steps for diagnosis and the proper course of treatment
for any condition or patient, including suitable and appropriate tests, 16 15 14 13 1 2 3 4 5 6 7 8 9 10
medications or medical devices to be used for or in conjunction with any
diagnosis or treatment.
Due to ongoing research, discoveries, modifications to medicines,
equipment and devices, and changes in government regulations, the in-
formation contained in this book may not reflect the latest standards,
developments, guidelines, regulations, products or devices in the field.
Readers are responsible for keeping up to date with the latest develop-
Editors and Other Contributors v

Editors
Paul A. Friedman MD, FACC, FHRS Samuel J. Asirvatham MD, FACC, FHRS
Consultant, Division of Cardiovascular Diseases Consultant, Divisions of Cardiovascular Diseases
Mayo Clinic, Rochester, Minnesota and Pediatric Cardiology
Professor of Medicine Mayo Clinic, Rochester, Minnesota
College of Medicine, Mayo Clinic Professor of Medicine and of Pediatrics
College of Medicine, Mayo Clinic
Melissa A. Rott RN
Heart Rhythm Services David L. Hayes MD, FACC, FHRS
Division of Cardiovascular Diseases Consultant, Division of Cardiovascular Diseases
Mayo Clinic, Rochester, Minnesota Mayo Clinic, Rochester, Minnesota
Professor of Medicine
Anita Wokhlu MD College of Medicine, Mayo Clinic
Fellow in Electrophysiology, Mayo School of Graduate
Medical Education
College of Medicine, Mayo Clinic, Rochester, Minnesota
Assistant Professor of Medicine, College of Medicine,
Mayo Clinic
vi Editors and Other Contributors

Contributors
Craig S. Cameron MD, FACC, Oklahoma Heart Institute, Tulsa, Oklahoma (Cases 52 and 53)
Gregory A. Cogert MD, FACC, Oklahoma Heart Institute, Tulsa, Oklahoma (Cases 52 and 53)
Connie M. Dalzell RN, Mayo Clinic, Rochester, Minnesota
Joseph J. Gard MD, College of Medicine, Mayo Clinic, Rochester, Minnesota
Michael Glikson MD, FACC, FESC, Leviev Heart Center, Sheba Medical Center, Tel Hashomer, Israel (Case 54)
Michael J. Hillestad RN, Mayo Clinic, Rochester, Minnesota
Nancy Y. Lexvold RN, Mayo Clinic, Rochester, Minnesota
Madhavan Malini MBBS, College of Medicine, Mayo Clinic, Rochester, Minnesota
Marjorie L. Martin RN, Mayo Clinic, Rochester, Minnesota
David A. Sandler MD, FACC, FHRS, Oklahoma Heart Institute, Tulsa, Oklahoma (Cases 52 and 53)
Matthew J. Swale MBBS, College of Medicine, Mayo Clinic, Rochester, Minnesota
K. L. Venkatachalam MD, Mayo Clinic, Jacksonville, Florida
Tracy L. Webster RN, Mayo Clinic, Rochester, Minnesota
Preface vii

The book that you hold in your hands, A Case-Based Approach to Pace- presented. In light of the growing use of pacemakers, defibrillators, and
makers, ICDs, and Cardiac Resynchronization: Questions for Examination resynchronization devices, we are confident that readers will find this
Review and Clinical Practice, is a compilation of our favorite teaching practical means of self-assessment and education useful. Although the
cases that were seen at or sent to Mayo Clinic. As our device practice has questions are designed in a multiple-choice format that may be particu-
grown, we have found that one of the best ways to remain current and larly useful for self-assessment for test-takers, they are not formally vali-
to educate incoming physicians and nurses is the review of interesting dated board questions. This book is for any individual who sees patients
“unknown” clinical cases. Consequently, we established a morning con- with implantable devices, or who will be taking an examination related
ference in 2008 for the purpose of presenting and discussing interest- to device management.
ing or uniquely educational cases. Since learners ranged from cardiology
fellows who were new to the device practice to experienced nurses and How to Use This Book
physicians, group discussion brought out facets of interest at all levels. The cases generally progress from simpler to more complex, understand-
Cases for this book were selected based on clinical relevance and their ing that there will be individual variation in what constitutes a difficult
usefulness for illustrating general principles, practical tips, or interesting case.
findings in device practice. Occasionally, manufacturer-specific features There is no table of contents because the case numbers are clearly
are discussed, but always with a goal of advancing general concepts in marked at the top of each page and we specifically did not want to
device management. include in the beginning of the book a listing of the “diagnosis” for each
The cases in this book are presented as a case history, an image case and therefore limit the ability for the reader to approach the cases
when pertinent, and a multiple-choice question. The answer and a as unknowns.
detailed explanation is presented on subsequent pages. We’ve adopted For the reader interested in reviewing a specific type of case (such
this format to encourage the reader to think through the differential as “T-wave oversensing” or “inappropriate shock”), two resources are
diagnosis and approach the clinical problem based on the information offered. An appendix is provided that identifies the major diagnostic
viii Preface

dilemma presented by each case, and the index will direct the reader to greatly from friends and colleagues at other institutions who have kindly
cases and discussions focusing on specific issues. However, we encour- shared interesting cases with us, and permitted us to include them in
age readers to progress sequentially through cases as unknowns to maxi- this work. We are grateful for their generosity. If you come across an
mize learning and interest. interesting case that you would like included in a future edition of this
This book is one of two volumes. The first volume includes intro- book, we would love to discuss it with you. E-mail addresses are listed
ductory and intermediate cases. The second volume includes additional below for that purpose. Please enjoy the cases! We look forward to your
intermediate cases as well as advanced cases. There are more multipart feedback and future contribution.
cases in volume 2, to delve more deeply into important concepts.
In various electronic versions of this book, hypertext links and —Paul Friedman MD and David Hayes MD
linked indices have been added to facilitate navigation. Also, a combined
index that covers both volumes is available at www.cardiotextpublishing
.com/titles/detail/9781935395812. Samuel Asirvatham: asirvatham.samuel@mayo.edu
This text includes a collective wisdom of numerous physicians, Paul Friedman: friedman.paul@mayo.edu
nurses, technicians, educators, and practitioners. We are indebted to the David Hayes: dhayes@mayo.edu
entire Heart Rhythm services team at Mayo Clinic for identifying and Melissa Rott: rott.melissa@mayo.edu
discussing cases, and educating us with them. We have also benefitted Anita Wokhlu: woklhu.anita@mayo.edu
Abbreviations ix

A atrial EP electrophysiological PVC premature ventricular


AF atrial fibrillation FFRW far-field R wave contraction
APC atrial premature contraction ICD implantable cardioverter- RAO right anterior oblique
AS atrial sensed defibrillator RBBB right bundle branch block
ASD atrial septal defect IV intravenous RV right ventricle; right ventricular
AT atrial tachycardia J Joules RVOT right ventricular outflow tract
ATP antitachycardia pacing LAO left anterior oblique SVT supraventricular tachycardia
AV atrioventricular LBBB left bundle branch block TARP total atrial refractory period
AVNRT atrioventricular nodal LV left ventricle; left ventricular TENS transcutaneous electrical
reentrant tachycardia LVEF left ventricular ejection fraction nerve stimulation
BBB bundle branch block MRI magnetic resonance imaging V ventricular
CI confidence interval OR odds ratio VA ventriculoatrial
CRT cardiac resynchronization PA pulmonary artery VF ventricular fibrillation
therapy PAC premature atrial contraction VRR ventricular rate regulation
CT computed tomographic PMT pacemaker-mediated tachycardia VS ventricular sensed
ECG electrocardiogram PVARB postventricular atrial VSD ventricular septal defect
EGM electrogram blanking period VT ventricular tachycardia
EMI electromagnetic interference PVARP postventricular atrial
refractory period
A Case-Based Approach
TO Pacemakers, ICDs, AND

Cardiac Resynchronization Volume 1

Questions for Examination Review and Clinical Practice


Case 1
RESYNCHRONIZATION
2 A 67-year-old male received a dual-chamber ICD for inducible ventricular tachycardia
PACEMAKERS, ICDs, AND CARDIAC
,

in the presence of ischemic heart disease. Four years later he received his first shock
during a visit to his chiropractor. He had sought chiropractic treatment due to chronic
shoulder discomfort.

Device settings:
• Mode: DDD
• Mode switch: on
• Pacing rate: 40 bpm (lower rate), 120 bpm (upper rate)
• Amplitude: 2.6 V (right ventricular), 2.0 V (atrial)
• Pulse width: 0.50 ms (right ventricular), 0.40 ms (atrial)
• Sensitivity: 0.18 mV (right ventricular), 0.18 mV (atrial)
• Dynamic AV: off
• Refractory after pace: 250 ms
• Antitachycardia therapies: initial burst ATP followed by 33-J shock

Upon interrogation the tracing in Figure 1.1 was obtained.


Friedman, Rott, Wokhlu, Asirvatham, Hayes 3

Figure 1.1  Tracing upon


interrogation.

Q
:
As a result of the EGMs and therapies delivered,
what would be your next step?
1. Initiate new or additional antiarrhythmic medications
2. Consider ablation of the ventricular ectopic focus
3. Reprogram ventricular sensitivity
4. Look for source of EMI
4 A Case-Based Approach to Pacemakers, ICDs, and Cardiac Resynchronization

1 4. Look for source of EMI


The tracing reveals sinus rhythm with a rate of approximately 52 bpm. was stopped and no further noise is noted. Diathermy is a method of
However, other signals are present on the EGMs, often occurring at a physical therapy that involves generating local heat in body tissues by
regular cycle length of approximately 220 ms, which, if consistent, would high-frequency electromagnetic currents.
be equivalent to a rate of approximately 272 bpm. These signals are A question could arise as to whether a problem with ventricular
sensed as ventricular events and labeled as VF or VT. Burst ATP therapy lead integrity was responsible for this appearance. When oversensing
is delivered at the end of the first strip and a 33-J shock is delivered after occurs as a result of loss of lead integrity, the VV intervals would most
the second tracing. Postshock tracing is shown in Figure 1.2. The signals likely be shorter and more irregular.
that were occurring at a cycle length of 220 ms are no longer present. Any circumstance in which a device patient is going to be subjected
Note that the tracing includes both AS-VP cycles as well as VS events to any equipment with known potential for electromagnetic interference
that appear to be premature ventricular contractions. should be carefully assessed prior to initiating therapy.
The appearance of very regular signals without any corresponding Answers 1 and 2 are incorrect because the abnormality noted is
intrinsic events on the EGM or surface recording suggests electromag- due to external EMI, not an intrinsic rhythm disturbance. Answer 3 is
netic noise or interference. In this example, the patient was receiving a incorrect because ventricular sensing is fine in the absence of the EMI.
treatment with diathermy. When the shock was delivered the diathermy
Friedman, Rott, Wokhlu, Asirvatham, Hayes 5

Figure 1.2  Postshock tracing.


Case 2
RESYNCHRONIZATION
6 An 83-year-old male had a newly implanted single-chamber pacemaker and right
PACEMAKERS, ICDs, AND CARDIAC
,

ventricular pace/sense lead when the chronic ventricular pace/sense lead was
damaged during recent cardiac surgery. He is pacemaker-dependent.

Device settings:
• Mode: VVIR
• Pacing rate: 60 to 120 bpm
• Ventricular lead: output 5.0 V at 0.4 ms; sensitivity 2.5 mV
• Pace/sense polarity: bipolar

The rhythm strip shown in Figure 2.1 was obtained following the procedure.
Friedman, Rott, Wokhlu, Asirvatham, Hayes 7

Figure 2.1  Rhythm strip.

Q
: What is the etiology of these multiple pacing spikes?
1. Artifact from hospital monitor
2. Ventricular safety pacing
3. Biventricular pacemaker with significant VV offset
4. Presence of temporary pacemaker undersensing permanent pacing
8 A Case-Based Approach to Pacemakers, ICDs, and Cardiac Resynchronization

2 4. Presence of temporary pacemaker undersensing


permanent pacing 24 hours, to be certain that the new lead is functioning normally, and
The permanent pacemaker was pacing and capturing at the programmed avoid any risk of ventricular asystole should the lead malfunction. Oth-
lower rate limit of 60 bpm (1000 ms) (Figure 2.2). The additional pacing ers believe that the potential complications and/or confusion of leaving
outputs are occurring at 30 bpm (2000 ms), which was the programmed another functioning lead in place outweighs the benefit. Still others pre-
parameter of the temporary pacemaker, still in place overnight due to fer to leave the temporary lead in place and connected to a temporary
the patient’s pacemaker dependency. When the newly placed permanent pacemaker but turned off and ready to turn on immediately if any ab-
pacing lead was assessed the thresholds were excellent, so the tempo- normalities are seen in the monitored patient.
rary pacemaker was programmed “off,” alleviating the additional pacing When checking devices post–cardiac surgery, consider:
spikes. • Changes in sensing, impedances, or threshold when leads
This is almost certainly not artifact because the cycle lengths were are purposefully moved, or inadvertently dislodged or
consistent, which would be highly unlikely for artifact, so answer 1 is damaged
incorrect. Answer 2 is incorrect because ventricular safety pacing is not • Temporary epicardial pacing leads that may or may not be
a feature in single-chamber pacemakers. This may have been a VVIR functioning, which could result in:
biventricular pacemaker, but the dual pacing spikes were not occurring • additional pacemaker artifacts (as seen previously)
with every paced beat because dynamic VV offsets are not available in • competition between permanent pacing and
contemporary CRT devices, so answer 3 is incorrect. temporary pacing
Some controversy exists regarding management of the pacemak- • inhibition of permanent pacemaker by temporary
er-dependent patient after implant. Some institutions/clinics prefer to pacing
leave a temporary pacing wire in place for a short period of time, eg,
Friedman, Rott, Wokhlu, Asirvatham, Hayes 9

Figure 2.2  Rhythm strip with added intervals.


Case 3
RESYNCHRONIZATION
10 An 86-year-old man has had a dual-chamber pacemaker implanted for the past 11 years
PACEMAKERS, ICDs, AND CARDIAC
,

due to intermittent high-grade AV block. He has a history of aortic valve replacement


and amyloidosis.

Device settings:
• Mode: DDDR
• Pacing rate: 70 to 120 bpm
• Paced AV delay: 150 ms
• Sensed AV delay: 120 ms
• Rate adaptive AV: on
Friedman, Rott, Wokhlu, Asirvatham, Hayes 11

Figure 3.1  Rhythm strip.

Q
:
What phenomenon is captured on the rhythm strip
in Figure 3.1?
1. Rate response accelerates rhythm and lengthens AV delay in response to activity
2. Programming head brief loss of communication with device
3. Ventricular safety pacing
4. Mode switch in response to an atrial arrhythmia
12 A Case-Based Approach to Pacemakers, ICDs, and Cardiac Resynchronization

3 2. Programming head brief loss of communication


with device telephonic rhythm strips, always be aware of both nonmagnet and mag-
The device is a Medtronic Preva model 7088. This generation of Med­ net rhythm strips to watch for the difference. A change in the expected
tronic pacemakers responded to programmer head application by initia- magnet response, usually a decrease in pacing rate, is indicative of bat-
tion of the magnet response. In this situation, the programming head tery depletion.
had slipped off during the interrogation and then was quickly reposi- Answer 1 is incorrect, for while rate response does increase the
tioned. Thus the beginning of the rhythm strip shows AV sequential rate and that increase in rate could result in a shortened AV delay, the
pacing at the lower rate limit of 70 bpm and rate-adaptive paced AV increase occurred in just one beat from 70 bpm immediately to 100
delay of about 150 ms (Figure 3.2). The brief loss of the atrial EGM oc- bpm, which is too fast for rate response to initiate. Answer 3 is incor-
curs when the programming head slips off and then reappears when it rect, for ventricular safety pacing would not change the rate, only the
is repositioned over the device. The magnet response for this Medtronic AV delay, in which case would be to 80 ms and not 100 ms. In addition,
device is asynchronous AV pacing of the first 3 beats at 100 bpm with the marker channel would show an additional marker for ventricular
a shortened AV delay of 100 ms, then to 85 bpm with the programmed safety pacing. Answer 4 is incorrect because the mode does not actually
paced AV delay of 150 ms. To stop the magnet rate, there is a “Cancel change; the device continues in DDDR mode and there is not a sudden
Magnet” button on the programmer, which must be selected to return flurry of A sensed events to trigger the mode switch.
to the regular programmed parameters.
Each company has its own specific magnet response with usual
increase in pacing rate and shortened AV delay. When reviewing trans-
Friedman, Rott, Wokhlu, Asirvatham, Hayes 13

Figure 3.2  The first 7 events are presenting rhythm of AP-VP with rate-adaptive AV delay. The next 3
events, the rate increases to 100 bpm with shortened AV delay of 100 ms. In the last 6 events the rate
decreases to 85 bpm with programmed paced AV delay of 150 ms.
Case 4
RESYNCHRONIZATION
14 An 81-year-old male presents for routine dual-chamber pacemaker interrogation
PACEMAKERS, ICDs, AND CARDIAC
,

(Figure 4.1). His device was implanted 6 years ago for sick sinus syndrome with
syncope. He also has a history of paroxysmal atrial fibrillation.

Device settings:
• Mode: DDDR
• Pacing rate: 55 to 130 bpm
• Mode switch: on at atrial detect rate of 175 bpm
• Paced AV delay: 180 ms; sensed AV delay: 150 ms
• Atrial lead parameters: 3.5 V at 0.4 ms and 0.25 mV
• Ventricular lead parameters: 2.5 V at 0.4 ms and 4.0 mV
Friedman, Rott, Wokhlu, Asirvatham, Hayes 15

Figure 4.1  Presenting rhythm.

Which of the following explains the apparent difference

Q
:
between the surface ECG and both the marker channel
and the ventricular EGM?
1. Device undersensing atrial flutter
2. Surface ECG displaying artifact
3. Noise reversion from TENS unit
4. Antitachy pacing being delivered
16 A Case-Based Approach to Pacemakers, ICDs, and Cardiac Resynchronization

4 2. Surface ECG displaying artifact


The surface ECG is displaying artifact. The patient has Parkinson’s dis- pacing mode programmed AAI at 90 bpm (Figure 4.3). The artifact was
ease, and associated tremors are causing the artifact. As the lower ECG eliminated by switching the leads to the abdomen.
limb leads are connected to his wrists, the surface ECG shows consistent Answer 1 is incorrect because the 12-lead ECG demonstrates
deflections that give the appearance of atrial flutter. Due to the patient’s AV sequential pacing with atrial capture; in addition moving the limb
history of paroxysmal atrial fibrillation, a 12-lead ECG was obtained leads eliminated the artifact. Answer 3 is incorrect because noise from
(Figure 4.2). This revealed AV sequential pacing at the lower rate limit a TENS unit would usually result in oversensing on the atrial channel
of 60 bpm with no evidence of atrial arrhythmia. Atrial capture is seen and an increase in sensed markers. Antitachycardia pacing would be de-
clearly in lead II (Figure 4.2) and during atrial threshold testing with livered at faster intervals than the artifact interval shown on the surface
ECG in Figure 4.1.

Figure 4.2  A12-lead ECG.


Friedman, Rott, Wokhlu, Asirvatham, Hayes 17

Figure 4.3  Atrial threshold testing (AAI 90 bpm) showing loss


at 1.5 V when consistent QRS complexes ceased.
Case 5
RESYNCHRONIZATION
18
PACEMAKERS, ICDs, AND CARDIAC
,

Figure 5.1  Tracing showing, from top to bottom, surface ECG,


atrial channel, ventricular channel, and markers.
Friedman, Rott, Wokhlu, Asirvatham, Hayes 19

Q
:
Which of the following best describes the tracing
in Figure 5.1?
1. Normal device function
2. Intermittent output failure
3. Atrial lead fracture
4. Inappropriate mode switch
20 A Case-Based Approach to Pacemakers, ICDs, and Cardiac Resynchronization

5 1. Normal device function


This tracing shows normal pacemaker function recorded during a ven- system. Since pacing stimuli are so brief in duration, they are not always
tricular threshold test. Figure 5.2 is annotated. The arrows labeled “1” captured by digital recording systems, which sample the analog signal. It
show a temporary loss of telemetry. Upon resumption of telemetry, a is clear that these complexes are paced, however, since their morphology
pacing threshold test is initiated by decrementing voltage amplitude at matches the neighboring complexes rather than intrinsic morphology.
a fixed pulse width. Note that the pacing rate is just slightly faster than After arrow 4 in the figure, the complexes appear fully paced. When
the instrinsic sinus rate. Each of the upward pointing arrows indicates a the voltage is decreased from 0.7 to 0.6 V, capture is lost, making the
pacing artifact. The first pacing artifact (arrow 2 in the figure) precedes threshold 0.7 V.
a pseudofusion complex. Because ventricular depolarization is nearly Answer 2 is incorrect because there is no evidence of intermittent
complete, the intrinsic QRS morphology is unchanged by the pacing output failure. During the threshold test the output voltage is gradually
stimulus. Since the pacing rate is faster than the intrinsic rate, there is decreased to define the capture threshold; this is normal. With failure
progressive fusion of intrinsic and paced depolarizations. With each suc- to output, pacing artifact is not present and there is no paced complex.
cessive complex, the paced wavefront captures an increasing proportion In this case, the intermittently absent pacing artifacts are due to a quirk
of ventricular tissue. Arrow 3 in the figure indicates where QRS fusion in telemetry, as confirmed by the paced QRS complex. Answer 3 is in-
is clearly apparent on the surface EGM. In the lower panel, as voltage correct because there are no make-break potentials or other findings to
output is reduced, some QRS complexes are no longer preceded by pac- suggest atrial lead fracture. Answer 4 is incorrect because there is no evi-
ing artifacts (see asterisk in figure). This is an artifact of the recording dence of a mode switch; the VVI mode was used for threshold testing.
Friedman, Rott, Wokhlu, Asirvatham, Hayes 21

Figure 5.2  A threshold test with a pacing rate that is close to the
intrinsic sinus rate, showing a ventricular threshold of 0.7 V.
Case 6
RESYNCHRONIZATION
22 A 52-year-old female undergoes dual-chamber pacemaker implantation for intermittent
PACEMAKERS, ICDs, AND CARDIAC
,

AV block. Right atrial and right ventricular leads are placed. Shortly after implantation
the ECG shown in Figure 6.1 is obtained.
Friedman, Rott, Wokhlu, Asirvatham, Hayes 23

Figure 6.1  Patient’s ECG after implantation.

Q
: What is the most logical first step after seeing this ECG?
1. Shorten the AV interval
2. Chest x-ray
3. MRI of chest
4. Reprogram atrial sensitivity
24 A Case-Based Approach to Pacemakers, ICDs, and Cardiac Resynchronization

6 2. Chest x-ray
The ECG demonstrates an RBBB, which usually indicates activation of the level of the atrial lead as opposed to being more inferior and cross-
the left ventricle. With a lead placed in the coronary sinus either inten- ing the tricuspid valve before being directed to the left. On the lateral
tionally or inadvertently, RBBB could be seen but the intention in this film the lead is directed posteriorly, consistent with a left ventricular
patient was to place the ventricular lead in the right ventricle. lead position.
RBBB could also be a result of the lead being placed across an When recognized acutely, the lead should be repositioned (Figure
atrial or ventricular septal defect (or patent foramen ovale) or by perfo- 6.3). Prior to doing so, it would be prudent to perform an ECG to deter-
ration of the right ventricle and advancement of the lead through the mine the location and size of the defect and whether there is any signifi-
pericardial space to the left ventricular surface. cant degree of right-to-left shunting. If significant right-to-left shunting is
A chest x-ray should be obtained to definitively determine the posi- present, the potential embolic risk of chronic transvenous pacing should
tion of the right ventricular lead. In the chest x-ray shown in Figure 6.2, be considered.
the lead has a high “take-off” on the PA film, ie, it crosses to the left at

Figure 6.2  Patient’s chest x-ray after implantation. Figure 6.3  Patient’s chest x-ray after repositioning
of the ventricular lead.
Friedman, Rott, Wokhlu, Asirvatham, Hayes 25

Following repositioning of the right ventricular lead, the ECG and present. Nothing would be accomplished by reprogramming any param- 6
chest x-ray shown in Figure 6.4 were obtained. On the chest x-ray the eters, answers 1 and 4.
“take-off” for the ventricular lead on the PA film is now considerably MRI, answer 3, was not necessary to make the diagnosis in this pa-
lower, and on the lateral film the lead is now directed anteriorly. tient, and generally MRI is a relative contraindication unless the patient
Other than a RBBB morphology of the paced QRS, no pacing has an MRI-resistant pacing system in place.
abnormalities are noted on the original ECG. Lead change artifacts are

Figure 6.4  Patient’s ECG and chest x-ray following repositioning.


Case 7
RESYNCHRONIZATION
26 A 75-year-old male received an ICD 7 years ago due to an out-of-hospital cardiac arrest
PACEMAKERS, ICDs, AND CARDIAC
,

and dilated cardiomyopathy. His second dual-chamber ICD, a Medtronic model 7271,
is currently implanted. The RV lead is a Medtronic model 6943-65 true bipolar lead
implanted with his initial ICD 7 years ago.

Device settings:
• Mode: DDDR
• Pacing rate: 70 bpm (lower), 120 bpm (upper)
• AV delay: 230 ms (paced and sensed)
• Amplitude: 2.0 V (atrial), 3.0 V (ventricular)
• Pulse width: 0.6 ms (atrial), 0.6 ms (ventricular)
• Sensitivity: 0.45 mV (atrial), 0.6 mV (ventricular)

Lead testing results (Figure 7.1):


• Atrial: P wave 2.4 mV, impedance 444 ohms, threshold 1.0 V at 0.4 ms
• Ventricular: R wave 9 mV, impedance 444 ohms, threshold 2.0 V at 0.3 ms
Friedman, Rott, Wokhlu, Asirvatham, Hayes 27

Figure 7.1  Presenting rhythm strip.

Q
: Why are there 2 “VS” markers for each QRS?
1. P-wave oversensing
2. T-wave oversensing
3. Double-counting of the QRS complex
4. Bigeminal PVCs
28 A Case-Based Approach to Pacemakers, ICDs, and Cardiac Resynchronization

7 1. P-wave oversensing
Markers show that the atrial lead first senses the P wave labeled AS, and Differential diagnosis for this phenomenon would include:
then the ventricular lead senses both the P wave and the R wave as VS. • RV lead dislodged to the tricuspid annulus or right atrium
Note that this oversensing of the P wave by the ventricular lead does not near the valve
occur with every intrinsic beat, as the fifth and sixth events are sensed • RV lead implanted in the coronary sinus
appropriately. This oversensing occurred at the programmed sensitivity • Lead-to-lead interaction with the atrial lead hitting the
of 0.6 mV (Figure 7.2). The device was then reprogrammed to 0.9 mV ventricular lead during valve motion
(Figure 7.3) and the far-field P-wave oversensing by the ventricular lead • RV lead distal coil crossing tricuspid valve, thus sensing
was not seen at this setting. A chest x-ray (Figure 7.4) showed normal RV atrial activity due to integrated bipolar versus true bipolar
lead placement and integrity. Close monitoring of this lead was advised. sensing; this is more common with proximally placed lead
Eventually, the oversensing occurred at even the least-sensitive program- or in children (with smaller hearts)
mable value of 1.2 mV, so the lead was capped and a new lead implanted. • RV lead insulation break near atrial lead allowing for
sensing of atrial activity

Figure 7.2  V sensitivity programmed at 0.6 mV.


Friedman, Rott, Wokhlu, Asirvatham, Hayes 29

Figure 7.3  V sensitivity reprogrammed to 0.9 mV.

Figure 7.4  Patient’s chest x-ray.


Case 8
RESYNCHRONIZATION
30 A 77-year-old woman was implanted with a dual-chamber pacemaker 3 months prior to
PACEMAKERS, ICDs, AND CARDIAC
,

device interrogation. Implant indication showed sinus node dysfunction with syncope.

Device settings:
• Mode: DDDR 60 to 130 bpm
• Mode switch: on at atrial tachycardia detection rate of 170 bpm
• Paced AV delay: 250 ms; sensed AV delay: 200 ms; search AV: on
• PVARP: 250 ms; rate-responsive PVARP high; PVAB 150 ms
• Atrial lead parameters: output 5.0 V at 0.4 ms; sensitivity: auto
• Ventricular lead parameters: output 5.0 V at 0.3 ms; sensitivity: auto
• Pace/sense polarity: bipolar

Diagnostics reveal 2189 mode switches occurring over the past 3 months since
implant. Total time of mode switching accounts for less than 1%, and the AT/AF
burden graph shows an overall time of 0%. Lead testing results in normal sensing,
impedance, and threshold values for both leads. Saved EGM for these episodes appears
in Figure 8.1
Friedman, Rott, Wokhlu, Asirvatham, Hayes 31

8
Figure 8.1  Patient’s EGM.
From top to bottom are
shown: atrial electrogram,
ventricular electrogram,
markers, and intervals.

Q
:
Which is the best explanation for these mode switch
occurrences?
1. Newfound paroxysmal atrial arrhythmia
2. Far-field R-wave oversensing
3. Retrograde VA conduction due to loss of atrial capture
4. Fractured atrial lead
32 A Case-Based Approach to Pacemakers, ICDs, and Cardiac Resynchronization

8 2. Far-field R-wave oversensing


Far-field R wave (FFRW) oversensing of the ventricular pacing complex tently following the ventricular pacing. Answer 4 is incorrect, for lead
was occurring by the atrial lead, resulting in numerous mode switches of fracture would result in frequent, varying signals of noise on the atrial
short duration. The beginning of the tracings shows A paced/V paced EGM, and the atrial EGM is very consistent. Also, lead testing values
events followed by an atrial event in refractory (marker AR). As the were all within normal range and consistent with testing from implant
mode switch (Trigger) occurs, the device stops atrially pacing and begins 3 months earlier.
to sense intrinsic atrial activity. At this point, the FFRW atrial events Initially, the atrial sensitivity was adjusted from auto adjusting to a
are now marked as AS, and the true sinus event is marked as AR. These set value of 0.5 mV, which alleviated the FFRW oversensing during that
FFRW events on the atrial EGM are almost as large as the sensed sinus visit. The patient then sent a routine remote monitoring transmission 3
events due to the close proximity of the atrial lead to the ventricle. months later, which revealed continued mode switching with now over
Answer 1 is incorrect, for Figure 8.1 shows varying AA intervals 35,000 episodes and an increase in ventricular pacing from 2% to 53%.
occurring after the mode switch (AMS) trigger, but consistent VA inter- EGMs showed AV dyssynchrony. Patient reported shortness of breath,
vals. Answer 3 is incorrect, for no retrograde VA conduction occurred weakness, and fatigue. As the FFRW oversensing continued to occur af-
by pacing VVI at 100 bpm. The surface ECG showed P waves continu- ter the sensitivity changes, the PVAB was increased from 150 to 170 ms,
ing at the intrinsic sinus rate of 60 bpm in complete dissociation with as shown in Figure 8.2. The patient returned for recheck in 1 month,
the ventricular pacing rate of 100 bpm. The atrial EGM showed larger, and only one mode switch had occurred over the past month since the
consistent deflections for the sinus rate, and smaller deflections consis- PVAB was increased.
Friedman, Rott, Wokhlu, Asirvatham, Hayes 33

Figure 8.2  ECG and EGM recording during PVAB reprogramming. From top to bottom are shown: surface
ECG lead II, markers, atrial unipolar tip electrogram, and ventricular unipolar tip electrogram.
Case 9
RESYNCHRONIZATION
34 An 85-year-old male is seen in hospital complaining of intermittent chest pounding/
PACEMAKERS, ICDs, AND CARDIAC
,

palpitations. A dual-chamber pacemaker had been implanted 6 years earlier for


second-degree AV block. The device had been implanted at another institution and no
additional details were available.

Device settings:
• Mode: DDDR
• Pacing rate: 60 to 110 bpm
• AV delay: 180 ms
• PV delay: 150 ms
• Dynamic AV delay: on
• PVARP: dynamic on; maximum 320 ms
• Ventricular output: 2.5 V at 0.4 ms
• Atrial output: 2.0 V at 0.4 ms

While on telemetry the patient complained of the same sensation of pounding/


palpitations. Figure 9.1 shows what the tracing revealed.
Friedman, Rott, Wokhlu, Asirvatham, Hayes 35

Figure 9.1  Patient’s telemetry tracing.

Q
:
Assuming that the patient should be seen, which of the
following would likely be the most appropriate action?
1. Assess and reprogram the rate-adaptive sensor parameters
2. Lengthen the PV interval
3. Lengthen the PVARP
4. Reprogram to DDIR
36 A Case-Based Approach to Pacemakers, ICDs, and Cardiac Resynchronization

9 3. Lengthen the PVARP


The tracing is consistent with pacemaker-mediated tachycardia (PMT). The event shown in Figure 9.3 was captured when the PMT ter-
Figure 9.2 shows the tracings obtained when the patient is seen in clinic. minated.
During the clinic assessment the patient again noted similar symp- In this tracing, AS represents an intrinsic atrial event that falls
toms. The event captured in Figure 9.2 demonstrates AS-VP followed by in a refractory period and is therefore not tracked. AS-VP cycles follow
premature atrial contraction (PAC), which could represent either a true termination of the PMT.
PAC or a sensed retrograde atrial depolarization. This in turn initiates PMT can be prevented by extending the PVARP or by turning on a
the AV interval, and a pacemaker-mediated tachycardia at the maximum PMT termination algorithm, if an option. Of the other options offered,
tracking rate occurs and causes the patient’s symptoms. this is not related to the rate-adaptive sensor, answer 1, and lengthening

Figure 9.2  Stored event retrieved during clinic visit. From top to bottom are shown: atrial electrogram, ventricular electrogram, surface ECG, and markers.
Friedman, Rott, Wokhlu, Asirvatham, Hayes 37

the PV interval, answer 2, would not reliably prevent PMT. Although ever, in the case, the APC altered AV synchrony and allowed retrograde 9
programming to DDIR, answer 4, would prevent PMT, it is less appropri- conduction that led to PMT.
ate for the patient because atrial tracking would no longer be possible. One clinical question would be whether the initial tracing repre-
Any event that disrupts AV synchrony could potentially initiate a sented an atrial tachycardia. If this question persists in the clinic, if ap-
PMT. The most common cause would be premature ventricular contrac- plication of a magnet during the tachyarrhythmia fails to terminate the
tions, and the second most likely cause is atrial failure to capture. How- tachycardia, then the etiology is not pacemaker-mediated.

Figure 9.3  PMT termination. From top to bottom are shown: atrial electrogram, ventricular electrogram, surface ECG, and markers.
Case 10
RESYNCHRONIZATION
38 A patient presents for a preoperative evaluation of his permanent pacemaker, which
PACEMAKERS, ICDs, AND CARDIAC
,

was implanted 21 months earlier and followed elsewhere.

Device settings:
• Mode: DDDR
• Pacing rate: 60 to 120 bpm
• Atrial output: 2.0 V at 0.4 ms
• Ventricular output: 2.5 V at 0.4 ms
• Sensitivity: A = 0.5 mV; V = 2.5 mV
• Accelerometer and minute ventilation sensors: on

Initial interrogation noted a single stored event of a high-rate episode (Figure 10.1).
The patient denied any symptoms; no awareness of palpitations or tachyarrhythmias.
Friedman, Rott, Wokhlu, Asirvatham, Hayes 39

10

Figure 10.1  Tracing from initial interrogation. From top to bottom are shown: atrial electrogram, ventricular electrogram, markers.

Q
:
Given this stored high-rate atrial episode,
what is your next step?
1. Program atrial sensitivity to a less-sensitive value
2. Activate mode switch
3. Turn off minute ventilation sensor
4. Attempt to correlate stored EGM with patient activities
40 A Case-Based Approach to Pacemakers, ICDs, and Cardiac Resynchronization

10 4. Attempt to correlate stored EGM


with patient activities There are few activity restrictions for device patients postimplan-
When carefully analyzed, the stored event correlates with a great deal tation. Pacemaker-dependent patients should probably forego welding
of “noise” on the tracing. It is difficult to be completely certain of all unless it is the source of their livelihood, in which case a more detailed
intrinsic ventricular events, but there are definite intrinsic events that investigation of their work site and evidence of interference must be
can be appreciated, as noted by the asterisks in Figure 10.2. (Identifica- done in a manner that determines what is safe for the patient and pro-
tion of intrinsic atrial events is not possible with any degree of certainty.) vides definitive guidelines or plans. For the non-pacemaker-dependent
The patient, a retired engineer, kept a detailed diary of his daily patient, care must still be taken. If the patient is a “hobby welder” and
activities and inquired during the device evaluation whether or not we uses equipment of ≤ 200 A, it may be possible for them to continue, but
could tell when exactly the event had occurred. Stored events are date- suggestions should be made as to how that could initially be done with
and time-stamped. When the patient was aware of when the high-rate some supervision by another person and/or monitoring during the activ-
episode had occurred, he immediately correlated it with a diary entry ity to be certain there is no significant electrical interference.
indicating that he had made a single and very brief attempt to demon- Before making any programming changes based on a stored
strate to his grandson how an arc welder was used. The arc welder was high-rate atrial event, ie, answers 1, 2, and 3, one must first determine
operating at an amperage of 250 A. He was otherwise standing behind whether it is physiologic and if there is any potential explanation for its
or away from his grandson while the welder was on. occurrence.
Friedman, Rott, Wokhlu, Asirvatham, Hayes 41

10

Figure 10.2  Stored event. From top to bottom are shown: atrial electrogram, ventricular electrogram, markers.
Case 11
RESYNCHRONIZATION
42 A 27-year-old woman received a pacemaker 2 weeks prior to the transtelephonic
PACEMAKERS, ICDs, AND CARDIAC
,

pacemaker check shown in Figure 11.1. Implant indication is high-grade AV block


following valve repair surgery.

Device settings:
• Mode: DDD
• Lower rate limit: 60 bpm
• Upper tracking rate limit: 110 bpm; upper sensor rate limit: 120 bpm
• Mode switch: on
• Paced AV delay: 180 ms; sensed AV delay: 150 ms; search AV: on
• Atrial lead parameters: output 5.0 V at 0.4 ms; sensitivity 0.5 mV
• Ventricular lead parameters: output 5.0 V at 0.52 ms; sensitivity 2.8 mV
Friedman, Rott, Wokhlu, Asirvatham, Hayes 43

11

Figure 11.1  Transtelephonic nonmagnet rhythm strip.

Q
:
What rhythm is captured on this nonmagnet
transtelephonic monitoring strip?
1. Upper rate limit behavior
2. Tracking of intermittent bursts of atrial tachycardia
3. Atrial undersensing
4. Intermittent ventricular failure to output
44 A Case-Based Approach to Pacemakers, ICDs, and Cardiac Resynchronization

11 1. Upper rate limit behavior


The device is tracking an atrial rhythm greater than the upper rate limit rates, the upper rate limits were reprogrammed to 140 bpm. The refer-
but less than the mode switch atrial detect rate. Since the pacemaker will ring cardiologist did not want a ventricular rate above 140 bpm, thus the
not violate a programmed upper rate limit with pacing, the PR interval mode switch detect rate was left at 150 bpm.
becomes progressively longer when the atrial rate exceeds the upper rate Answer 2 is incorrect, for there is a consistent pattern to the
limit until a complex is “dropped,” since another P wave occurs before rhythm strip with one long VV interval followed by 3 shorter VV inter-
the next paced beat is scheduled. This phenomenon, shown in Figure vals occurring at the upper tracking rate limit of 110 bpm. There is no
11.2, is pacemaker Wenckebach. irregularity to the pattern, suggesting sinus dysrhythmia. Answer 3 is in-
The device was interrogated, and diagnostics revealed 97.7% of correct, for if atrial undersensing were occurring, either atrial pacing at
events were atrial (A) sensed/ventricular (V) paced. Histograms showed the lower rate limit or sensor-driven atrial pacing would occur, and there
atrial rates distributed from 60 to 140 bpm and ventricular rates from is no atrial pacing seen. When the P waves are visualized after the longer
60 to 120 bpm. Seventy-two mode switches occurred, accounting for a VV intervals, they are clearly tracked by ventricular pacing. Answer 4 is
total mode switched time of 2.7%. Stored EGMs of these mode switch incorrect, for the majority of the ventricular pacing occurs at the pro-
episodes showed 2:1 A:V conduction of atrial rates from 150 to 170 bpm. grammed upper rate limit of 110 bpm, thus suggesting this programmed
To minimize upper rate limit behavior and appropriately track sinus upper rate limit is limiting the VV pacing interval.
Friedman, Rott, Wokhlu, Asirvatham, Hayes 45

11

Figure 11.2  Pacemaker Wenckebach phenomenon.


46
,
PACEMAKERS, ICDs, AND CARDIAC
RESYNCHRONIZATION

Figure 12.1  Surface ECG.


Case 12
Friedman, Rott, Wokhlu, Asirvatham, Hayes 47

12

Q
:
What does the tracing in Figure 12.1 include?
1. Intrinsic beat
2. Paced beat
3. Fusion beat
4. Pseudofusion beat
5. All of the above
48 A Case-Based Approach to Pacemakers, ICDs, and Cardiac Resynchronization

12 5. All of the above


This question tests your ability to identify intrinsic, fusion, pseudofu- and may have a LBBB morphology if it arises from the right ventricle.
sion, and pace capture ventricular complexes on a surface ECG. All ele- In addition, the depolarization is preceded by a pacemaker output pulse,
ments are seen, as annotated in Figure 12.2. which appears as a spike or artifact on ECG.
An intrinsic beat is a depolarization of any chamber that occurs A fusion beat is a hybrid beat that arises when ventricular depo-
without pacing. Intrinsically conducted ventricular beats may be narrow larization arises from 2 separate initiation sites and merges. Intrinsic
if they arise from the native conduction system or wide if they represent conduction results in partial depolarization and occurs nearly simul-
premature ventricular contractions. taneously with depolarization from the pacemaker output. A stimulus
A paced beat arises from pacing without an intrinsic contribution artifact is seen early in the complex, and the morphology of the fusion
and captures that chamber. A paced ventricular beat is typically wide, beat appears different from either intrinsic or paced alone. The mor-

Figure 12.2  Annotated surface ECG.


Friedman, Rott, Wokhlu, Asirvatham, Hayes 49

phology will vary depending on the relative contribution of each type one chamber is superimposed on a deflection arising from the depolar- 12
of depolarization. To make the diagnosis of a fusion beat, it is helpful ization of the other chamber.
to have a template of the intrinsic beat or a paced beat morphology for What is the clinical relevance of these findings? First, it is impor-
comparison. tant to recognize that fusion and pseudofusion beats can be normal phe-
Pseudofusion arises when a stimulus artifact is superimposed on nomena. Clear fusion confirms capture. However, when pseudofusion
an intrinsic P wave or QRS complex, giving the false appearance of cap- or questionable fusion occurs, it may be difficult to determine if the
ture. Although the stimulus artifact is present, the morphology of the P device failed to sense intrinsic or if the stimulus failed to cause depo-
wave or QRS is unchanged from the intrinsic morphology. The pacing larization. In some cases, it is helpful to confirm capture by increasing
stimulus is ineffective because it has been delivered when the tissue is the pacing rate to assess for a morphology consistent with capture and
refractory. to perform a capture threshold test. It is also important to confirm sens-
Pseudo-pseudofusion, not shown, occurs in the setting of dual- ing, which can be done by decreasing or inhibiting the basal pacing rate.
chamber pacing (most often DVI mode) when a stimulus artifact from
Case 13
RESYNCHRONIZATION
50 A question of pacemaker malfunction has arisen in a patient with a pacemaker
PACEMAKERS, ICDs, AND CARDIAC
,

programmed to DDDR, 60 to 140 bpm, with paced AV delay at 220 ms. You are called
to evaluate the rhythm strip shown in Figure 13.1. In some cycles, 2 pacing artifacts
are seen, but in other cycles, there are no pacing artifacts.
Friedman, Rott, Wokhlu, Asirvatham, Hayes 51

13

Figure 13.1  Patient’s rhythm strip.

During the cycles when 2 artifacts are seen, the

Q :
intervening QRS complex (circled in Figure 13.1) occurs
in which portion of the device timing cycle?
1. Ventricular blanking period
2. Crosstalk sensing window
3. Alert period
4. Postventricular atrial blanking period
52 A Case-Based Approach to Pacemakers, ICDs, and Cardiac Resynchronization

13 1. Ventricular blanking period


The timing cycle is detailed in Figure 13.2. The figure highlights an the ventricular channel is “blanked” to potential crosstalk from the
intrinsic QRS and a QRS preceded by a stimulus artifact. Although the atrial channel, ie, the “intrinsic deflection” of the intrinsic event occurs
actual R-R interval measures 800 ms, the lower rate limit defines a VV within the blanking period; (5) an intrinsic QRS is not sensed; and, ulti-
interval of 1000 ms (60 bpm). The second stimulus artifact, ie, the one mately, (6) a ventricular pacing stimulus is delivered dangerously close to
in front of the T wave, times out to 1000 ms after the native QRS com- the T wave. One potential consequence of this type of functional under-
plex, suggesting that this artifact is derived from the ventricular pacing sensing would be ventricular arrhythmia due to an R-on-T phenomenon.
stimulus. The other preceding artifact times out to exactly 220 ms (the One solution in this case would be to shorten the ventricular
duration of the AV delay) in front of the ventricular pacing artifact. This blanking period with the hope that the intrinsic QRS would then fall
suggests that other artifact corresponds to an atrial pacing stimulus— within the crosstalk sensing window.
even though it precedes a native QRS complex. Answer 2 is incorrect because detection of the QRS within the
The difference between the current R-R and the AV delay is 780 crosstalk sensing window would have likely led to a ventricular safety
ms. This time difference is referred to as the VA interval, or the atrial pacing response. Answer 3 is incorrect because sensing of the QRS oc-
escape interval, and reflects the duration of time that transpires after the curs with the ventricular alert period. Depending on pacing mode, this
ventricle is sensed or paced before atrial pacing will occur. can trigger an inhibited or triggered response. Answer 4 is incorrect
The sequence of events now becomes clear: (1) the intrinsic QRS because the postventricular atrial blanking period is the blanking of
is sensed; (2) the VA interval transpires; (3) an atrial pacing stimulus is the atrial channel after a ventricular activation; the QRS occurs after an
delivered; (4) the postatrial ventricular blanking period begins so that atrial pacing stimulus.
Friedman, Rott, Wokhlu, Asirvatham, Hayes 53

13

Figure 13.2  Rhythm strip annotated with timings.

Q :
Which term is appropriate for this ECG appearance
(circled in Figure 13.1)?
1. Fusion beat
2. Pseudo-pseudofusion beat
3. Pseudofusion beat
4. Safety pacing
54 A Case-Based Approach to Pacemakers, ICDs, and Cardiac Resynchronization

13 2. Pseudo-pseudofusion beat
This case tests your understanding of pacemaker timing and pseudo- Some form of pseudofusion or pseudo-pseudofusion appears to be
pseudofusion. going on. To make the distinction, you need to discern whether the
In Figure 13.1, all the QRS complexes are narrow, consistent with artifact that precedes the QRS is derived from the ventricular chamber
intrinsic conduction. Every alternate QRS complex is preceded by stimu- or the atrial chamber. When a pacing artifact from a given chamber fails
lus artifact without any change in QRS morphology in 2 leads. In addi- to contribute to depolarization because the chamber is refractory from
tion, every T-wave complex is preceded by a second stimulus, giving the an intrinsic depolarization, that is pseudofusion. When a pacing arti-
appearance of attempted dual-chamber pacing. fact arising from another chamber coincidentally precedes an intrinsic
Friedman, Rott, Wokhlu, Asirvatham, Hayes 55

depolarization, producing the same appearance of pacing noncapture Answer 3 is less likely because the pacing artifact in front of the T wave 13
coincident with intrinsic depolarization, that is pseudo-pseudofusion. is timed to 1000 ms from the previous QRS complex, suggesting they
Distinguishing the 2 would be simple if channel markers were are ventricular; conversely, the artifact in front of the QRS is timed 220
available. In the absence of such information, one needs to march out ms in front, suggesting that the atrial pacing artifact precedes the QRS.
pacemaker timing cycles. Answer 4 is incorrect because ventricular safety pacing usually occurs
Answer 1 is incorrect because although there is a pacing stimulus after crosstalk, which is not seen here. In addition, the safety pacing
preceding the QRS, the QRS morphology is no different than intrinsic. stimulus will usually occur with a very short AV delay (approximately
100 ms) rather than a long AV delay.
Case 14
RESYNCHRONIZATION
56 A 14-year-old female presents with intermittent, symptomatic complete heart block
PACEMAKERS, ICDs, AND CARDIAC
,

and a DDDR pacemaker is placed. The pacemaker results in complete resolution of her
symptoms. Two years later she develops recurrent symptoms of light-headedness. At
this time she is essentially pacemaker-dependent. She notes that it is not as profound
as it was at the time the original diagnosis was made.

Pacemaker interrogation reveals the programmed parameters are identical to what had
been recorded at the time of her most recent visit to the pacemaker clinic. On testing,
the pacemaker threshold was 0.5 V at 0.5 ms, below which there was complete loss of
ventricular capture. However, when the rhythm strip was observed for longer periods
of time at 0.5 V and 0.5 ms the abnormality in the tracing in Figure 14.1 was noted.
Even when programmed to the highest programmable output values, the tracing
appeared identical. Ventricular impedance was checked repetitively with the patient
sitting quietly, with manipulation of the pacemaker pocket, and with the patient
performing various arm maneuvers. The impedance remained in the range of 550 to
600 ohms on all checks.

A chest x-ray was obtained and with close inspection no abnormalities of the pulse
generator or leads could be detected.
Friedman, Rott, Wokhlu, Asirvatham, Hayes 57

14

Figure 14.1  Patient’s tracing.

Q
: What is the most appropriate clinical approach?
1. Program to maximum voltage amplitude and pulse width and reevaluate in 1 month
2. Place a new epicardial ventricular lead
3. Place a new ventricular lead and avoid future competitive sports
4. Place a new ventricular lead via alternate venous route
58 A Case-Based Approach to Pacemakers, ICDs, and Cardiac Resynchronization

14 4. Place a new ventricular lead via


alternate venous route initially in the same range as noted previously, but with slight tension on
Despite recording a normal impedance on the ventricular lead, with and the lead the impedance measured > 3000 ohms. A new lead was placed
without arm maneuvers and pocket manipulation, and despite normal via the axillary vein in hopes of avoiding a recurrent problem, since the
radiographic appearance, in the absence of a lead dislodgment, one must problem described was felt to be due to subclavian crush.
assume that the integrity of the ventricular lead is in question. In this The course of the axillary vein is such that the subclavius muscle
case, the patient noted that her recurrent symptoms had been primarily is avoided as is impingement between the clavicle and first rib, and the
during exercise. For that reason a treadmill exercise test was performed. risk of “crush” is minimized. Another option for this patient would be
In the tracing shown in Figure 14.2, intermittent ventricular failure to epicardial lead placement (answer 2), although this more invasive proce-
capture is again noted during the faster rates achieved during exercise. dure would carry additional morbidities.
It should be noted that this patient was a talented basketball player In the presence of intermittent failure to capture even at the high-
and on track to receive a full college scholarship to play the sport. The est outputs, reprogramming, answer 1, would not be safe. No long-term
original lead had been placed via the subclavian vein, and subclavian restrictions were placed regarding her athletic endeavors, answer 3, and
crush was suspected as the cause of the lead problems. in most situations every attempt should be made to allow to continue
When the patient underwent placement of a new ventricular lead, those activities that are important to their livelihood and/or their qual-
direct measurement of impedance on the chronic ventricular lead was ity of life.
Friedman, Rott, Wokhlu, Asirvatham, Hayes 59

14

Figure 14.2  Patient’s tracing during exercise.


Case 15
RESYNCHRONIZATION
60 A 27-year-old woman had a Fontan procedure, with modification of the Fontan by
PACEMAKERS, ICDs, AND CARDIAC
,

conversion to a lateral tunnel implanted with a single-chamber AAI pacemaker to


manage atrial arrhythmias after several failed attempts at repeat ablation for atrial
flutter.

Device settings:
• Mode: AAIR
• Pacing rate: 60 to 140 bpm
• Atrial lead parameters: output 5.0 V at 0.4 ms; sensitivity 0.3 mV
Friedman, Rott, Wokhlu, Asirvatham, Hayes 61

15

Figure 15.1  Patient’s rhythm strip.

Q
:
What is the best explanation for the rhythm shown in
Figure 15.1?
1. Atrial output programmed below threshold
2. P wave diminished and sensitivity needs adjusting
3. Atrial lead dislodgment
4. Undersensing of atrial tachycardia
62 A Case-Based Approach to Pacemakers, ICDs, and Cardiac Resynchronization

15 3. Atrial lead dislodgment


The 12-lead ECG indeed shows both atrial undersensing and atrial fail- It was felt that this patient would benefit from a pacemaker with atrial
ure to capture. The patient’s underlying rhythm is sinus bradycardia with antitachycardia pacing capabilities. The Medtronic EnRhythm model
LBBB at 55 bpm. Device interrogation revealed there was no sensing at P1501DR, available only as a dual-chamber pacemaker, was felt to be
the most sensitive setting and no capture at maximum output. The atrial the best device for this patient. Due to the patient’s coronary anatomy
lead had dislodged within a few short hours after the implant. This can and prior Fontan procedures an endocardial ventricular lead could not
be seen in the chest x-rays in Figure 15.2 and 15.3. The patient was also be implanted. An epicardial ventricular lead could be implanted and
experiencing intermittent chest wall stimulation following the implant. connected to the device, but clinically the patient had no absolute need
The device was reprogrammed to OAO and lead revision planned for for ventricular pacing support and it was felt that the potential morbidity
the next day. from a procedure to place an epicardial lead could not be justified.
The chest x-ray demonstrated a dual-chamber pacemaker with a The Medtronic EnRhythm model P1501DR requires ventricular
single atrial lead implanted and a plug inserted into the ventricular port. events, either sensed or paced, in order to treat atrial arrhythmias, be-

Figure 15.2  Patient x-ray of lead dislodgment. Figure 15.3  Lateral x-ray of lead dislodgment.
Friedman, Rott, Wokhlu, Asirvatham, Hayes 63

cause it must accumulate evidence for the atrial tachyarrhythmia. To can be used toward the criteria for an atrial tachyarrhythmia. With the 15
meet the criteria for an atrial tachyarrhythmia, the device must see more ventricular output programmed to minimal values, the drain on the bat-
atrial sensed events than ventricular events. If the device is programmed tery should be negligible. With the sensitivity at the least-sensitive set-
AAIR with no ventricular lead in place, it will sense the rapid atrial ting, any noise should be eliminated.
events above the atrial detection rate and annotate them as “TS” for Caution should be taken when the device is nearing the elective
“Tachy Sense,” but it will not treat these events because there is no ven- replacement indicator because this Medtronic pacemaker will switch
tricular event to compare them with, so detection is never met. If the from the programmed mode to the usual elective replacement indicator
device is programmed to AAIR with a ventricular lead implanted, the V parameters of VVI at 65 bpm. A patient who was reliant on the atrial
sensed events will count toward meeting the criteria for an atrial tachyar- pacing would be left without any pacing support.
rhythmia and treatment will occur. In this case, when no V lead is im- Although this was felt to be the option for this particular patient
planted, the device was programmed to either DDDR or DDIR, with the and in her best interest, and documented as such in the medical record,
ventricular outputs at the lowest programmable values and the sensitiv- the configuration utilized would be considered an off-label use of the
ity at the least-sensitive value so the “pseudo” ventricular paced events device.
Case 16
RESYNCHRONIZATION
64 A 73-year-old man with ischemic cardiomyopathy, QRS duration of 120 ms, and
PACEMAKERS, ICDs, AND CARDIAC
,

reduced ejection fraction undergoes Medtronic single-chamber defibrillator


implantation for secondary prevention after sudden cardiac arrest. Now, 3 years
later, he presents with multiple shocks over the last 24 hours. There are no signs
or symptoms of congestive heart failure, and his potassium level is 5 mmol/L. His
interrogation reveals 25 shock episodes for detected ventricular fibrillation. A
representative interval plot and EGM are shown in Figures 16.1 and 16.2, respectively.

Figure 16.1  Representative interval plot.


Friedman, Rott, Wokhlu, Asirvatham, Hayes 65

16

Figure 16.2  Patient’s EGM.

Q
: What is the most likely cause of these shocks?
1. Lead fracture
2. P-wave oversensing
3. T-wave oversensing
4. R-wave double-counting
66 A Case-Based Approach to Pacemakers, ICDs, and Cardiac Resynchronization

16 3. T-wave oversensing
This case illustrates the problem of inappropriate shocks for T-wave over- multaneous, alternating sets of VV intervals. The top row of dots sug-
sensing in the setting of a relatively small sensed R wave. gests a rate of approximately 600 ms (100 bpm). The lower row of dots
The first step is to recognize that the shocks are inappropriate. suggests a ventricular rate of 250 ms (240 bpm). Physiologically, such
The interval plot in Figure 16.1 provides the first clue. There are 2 si- alternation seems unlikely for ventricular tachycardia or ventricular fi-

Figure 16.3  EGM and corresponding sensing


on the marker channel.
Friedman, Rott, Wokhlu, Asirvatham, Hayes 67

brillation. This observation is sometimes referred to “tramtracking” or notice the small R-wave amplitude on the near-field. On the near-field, 16
“railroad tracks” and is consistent with double-counting or oversensing R waves are much smaller in amplitude overall—often less than 1 mV.
of alternating intervals (in this case, the QRS–T-wave and T-wave–QRS In fact, there is phasic variation to R-wave size (perhaps due to respira-
intervals). tion). As R waves diminish, T waves become more prominent. Recall
The next step is to determine the source of oversensing. It is im- that ventricular sensing is beat-to-beat function of Decay Delay curves
portant to look at all available EGMs to determine if P-wave oversensing, that depend on the size of the preceding R wave (as illustrated in Figure
R-wave double-counting, or T-wave oversensing is contributing to the 16.3). The diminished R to T ratio impairs the ability of the device to
inappropriate assessment of ventricular rates. “ignore” the T wave. Instead, the T wave is miscounted as a second R
Reviewing the EGMs and the corresponding sensing on the mark- wave. This is reported on the marker channel as ventricular fibrillation
er channel on Figure 16.3 clarifies the etiology of the abnormal interval detect followed by ventricular fibrillation sense (“FS”).
plots. The high-voltage or shock, far-field EGM is similar to a surface Answer 1 is incorrect because irregular R-R intervals and high-
EKG because it uses widely spaced, large electrodes (can/proximal coil frequency noise on the near-field EGM known as make-break potentials
and RV coil) and, therefore, can be used to identify P, QRS, and T waves. (often nonphysiologic, without QRS correlate) would be seen on the
In this case, the underlying rhythm is regular and normal in rate (600 near-field EGM. Answer 2 is incorrect because the double-counting was
ms or 100 bpm). The sensing, or near-field EGM is different. Acquisition not occurring at the time of expected P waves. Answer 4 is incorrect,
of signal comes from RV tip to ring, and therefore, only ventricularly because the R wave was not particularly wide (QRS duration 120 ms)
based signals are typically seen. Note that the scales on the near-field and and the double-counting occurred on the T wave.
far-field are automatically gained differently such that one might fail to

Q :
What is the next best step to manage these
inappropriate shocks?
1. Change the ventricular sensitivity threshold from 0.3 to 1.0 mV and repeat VF induction
2. Increase ventricular blanking following ventricular pacing
3. Force constitutive ventricular pacing to reduce T-wave amplitude
4. Lead revision
68 A Case-Based Approach to Pacemakers, ICDs, and Cardiac Resynchronization

16 4. Lead revision
Only revision of the ventricular lead or certain programming changes antitachycardia pacing, but not in inappropriate shocks. If the R wave is
(not listed among the answers) would be appropriate in this case. larger than the T wave, reprogramming can generally eliminate T-wave
T-wave oversensing can be categorized based on the near-field oversensing. However, if the R-wave amplitude is small (generally less
EGM as occurring (1) only during ventricular pacing, (2) with a large than 3 mV, although no specific value has been defined), reprogram-
R wave that is larger than the T wave (with R-wave amplitude >>T-wave ming is more difficult and increases the risk of undersensing. In our ex-
amplitude), or (3) with a small R wave that is smaller than the T wave. perience, programming options such as raising the minimum threshold
When T-wave oversensing occurs only following paced beats, the sensitivity value (ie, so-called decreasing the sensitivity, all manufactur-
pacing rate may be slowed (since sensing occurs off of the T wave rather ers) or adjusting Decay Delay (St. Jude Medical) are more effective when
than the QRS complex). This may result in bradycardia or ineffective the R to T ratio is greater than one. It has been suggested that stability
Friedman, Rott, Wokhlu, Asirvatham, Hayes 69

can be turned on if there is sufficient difference in the RT and TR inter- sensing incurs the risk of undersensing of R waves and possibly ven- 16
vals (so that therapy in the VT zone is inhibited due to interval variabil- tricular fibrillation. Increasing the ventricular blanking after ventricular
ity). However, when the R wave is smaller than the T wave, lead revision paced events (answer 2) might mitigate the T-wave oversensing that oc-
provides the most robust solution. Replacement with a true bipolar lead curs with pacing but will not affect T-wave oversensing of ventricular
(as opposed to an integrated bipolar lead) may minimize oversensing sensed QRS complexes. Forced ventricular pacing (answer 3) is not the
risk. Alternatively, a separate pace-sense lead can be added, as was done best answer. Although it can alter and reduce T-wave morphology and af-
in this patient. fect refractory periods, chronic RV pacing can negatively impact cardiac
Decreasing sensitivity (answer 1) is not ideal because the T wave is function in this patient with low ejection fraction. Additionally, prema-
larger than the R wave. Adjusting sensitivity to eliminate T-wave over- ture atrial or ventricular complexes may interrupt pacing and permit
oversensing to occur.
Case 17
RESYNCHRONIZATION
70 The same 73-year-old man from case 16, with a history of single-chamber defibrillator
PACEMAKERS, ICDs, AND CARDIAC
,

implantation, has a RV pace-sense lead added because of T-wave oversensing on


his defibrillator lead, which was causing inappropriate shocks. One month later, the
patient receives another defibrillator shock and remotely transmits his data. The
patient is currently in sinus rhythm. Figure 17.1 shows the shock episode.
Friedman, Rott, Wokhlu, Asirvatham, Hayes 71

17

Figure 17.1  Arrhythmia episode. The top row demonstrates far-field signal
derived from the high-voltage EGM (shock). The bottom row demonstrates
near-field signal derived from a separate RV sense-pace lead.

Q
: What is the most likely cause of the shock episode?
1. Lead dislodgment
2. Dual atrial and ventricular tachycardias
3. Electromagnetic interference
4. Oversensing of diaphragmatic myopotentials
72 A Case-Based Approach to Pacemakers, ICDs, and Cardiac Resynchronization

17 1. Lead dislodgment
This case illustrates the problem of inappropriate shocks from ventricu- with ventricular oversensing of atrial fibrillation. Administration of
lar oversensing of atrial signal due to ventricular lead dislodgment. a defibrillatory shock clears up much of the high-frequency signal on
Comparison of the near-field and far-field channels in Figure 17.2 the near-field, likely terminating the atrial fibrillation. A wide complex
clarifies the problem. Recall that the far-field signal is derived from the rhythm is then seen on the far-field, and irregular complexes (probably
shock circuit (pulse generator can/superior vena cava coil and RV coil) atrial in origin) are sensed on the near-field. Most likely, the rhythm is
and provides morphology information. The near-field EGM, in this case, an idioventricular rhythm (as the ventricular rate exceeds the atrial rate),
is derived from a separately implanted RV pace-sense lead, and is used by although intermittent ventricular sensing of a slowed atrial dysrhythmia
the device to identify and count ventricular signals. with aberrancy or postshock morphology changes cannot be excluded.
The far-field channel shows irregular, but distinct, R-R intervals These findings are consistent with dislodgment of the ventricular
and no apparent P waves, which is consistent with atrial fibrillation. lead into or near the atrium, resulting in inadvertent misclassification of
The near-field signal, in contrast, looks erratic and is also consistent atrial fibrillation as ventricular fibrillation. Figure 17.3 shows the real-

Figure 17.2  Arrhythmia episode, annotated.


Friedman, Rott, Wokhlu, Asirvatham, Hayes 73

time EGM that was transmitted following the shock. The ventricular ventricular signals. Atrial arrhythmias, in particular, may be sensed in- 17
EGMs on the near-field channel correspond to P waves rather than QRS correctly as ventricular arrhythmias.
complexes, indicating that the ventricular pace-sense lead was dislodged There is no ventricular tachycardia present, making answer 2 in-
to the atrium. The smaller, dedicated pace-sense lead may be more prone correct. Electromagnetic interference (answer 3) is incorrect because
to dislodgment into the atrium than a defibrillator’s lead. it would be present, and typically larger in amplitude, on the far-field
Oversensing of atrial signals on the ventricular lead can occur with EGM as well as the near-field EGM. Answer 4 is incorrect because dia-
dislodgment of the ventricular lead to the annulus or to the atrium (as in phragmatic myopotentials, although also near-field, present as bursts of
the present case). Integrated bipolar leads, particularly with proximally lower-amplitude, high-frequency noise that varies with respiration and
placed leads or in children, may also lead to oversensing of atrial EGMs occur most commonly after pacing or long diastolic intervals. The pat-
since the large distal coil is part of the sensing circuit. If it is placed at tern should be similar after a shock.
or near the level of the tricuspid valve annulus, it may record atrial and

Figure 17.3  Real-time EGM.


Case 18
RESYNCHRONIZATION
74 A 71-year-old woman with near syncope is found to have sinus node dysfunction and
PACEMAKERS, ICDs, AND CARDIAC
,

receives a dual-chamber pacemaker. The next day, the telemetry strip shown in Figure
18.1 is recorded.
Friedman, Rott, Wokhlu, Asirvatham, Hayes 75

18

Figure 18.1  Hospital telemetry following dual chamber pacemaker implantation.

Q
: What does this tracing most likely represent?
1. Nonsustained VT
2. Atrial fibrillation with aberrant conduction
3. Normal pacemaker function
4. Pacemaker mediated tachycardia
76 A Case-Based Approach to Pacemakers, ICDs, and Cardiac Resynchronization

18 3. Normal pacemaker function


The tracing shown in Figure 18.1 has been annotated in Figure 18.2. pacemaker is no longer in a tracking mode, and the underlying atrial
The tracing shows 2 surface ECG leads. At the left the rhythm is atrial tachyarrhythmia is seen with native conduction.
pacing with intrinsic conduction to the ventricle. The asterisks indicate Answers 1 and 2 are incorrect, since the wide-complex ventricular
premature atrial complexes. Note the ventricular pacing spike, labeled events seen in this telemetry strip are due to ventricular pacing and not
pseudofusion (Figure 18.1). Since a premature atrial complex may have ventricular tachycardia or atrial fibrillation with aberrancy. There are
a prolonged PR interval, the AV interval times out and a ventricular clearly ventricular pacing spikes at the beginning of the wide-complex
pacing pulse is delivered. However, at the same time there is an intrin- ventricular events, and this pacing is occurring within the programmed
sic ventricular complex. If the resultant complex is partially paced and parameters. This is not an example of PMT, answer 4, since the tachycar-
partially intrinsic, it is called a fusion beat. If it is entirely activated by dia begins after a captured atrial paced/ventricular sensed event. PMT
the His-Purkinje system, and the pacemaker spike doesn’t actually acti- occurs when retrograde conduction follows ventricular pacing, and the
vate the ventricles (so that the morphology matches intrinsic beats), it is sensed retrograde “A” leads to a paced “V,” which in turn is conducted
called pseudofusion. Next, there are 8 ventricular paced complexes that back to the atrium. The left-hand side of the tracings shows an atrial ar-
are irregular, during which the pacemaker is tracking an atrial arrhyth- rhythmia, not atrial sensed events due to retrograde conduction.
mia. Following the 8 complexes, a mode switch event occurs, so that the
Friedman, Rott, Wokhlu, Asirvatham, Hayes 77

18

Figure 18.2  Same tracing as Figure 18.1, annotated.


Case 19
RESYNCHRONIZATION
78 A 54-year-old man with a history of atrial flutter and symptomatic sinus pauses (up to
PACEMAKERS, ICDs, AND CARDIAC
,

7 seconds in duration) undergoes dual-chamber pacemaker implantation. At implanta-


tion, atrial and ventricular
lead parameters were nor-
mal when directly mea-
sured through the pacing
system analyzer. However,
the morning after implan-
tation, the R wave mea-
sured only 2.7 mV. Parame-
ters were optimized to im-
prove R-wave sensing. The
device was programmed to
DDDR mode. The patient
returns for 1-month follow-
up. An excerpt from his ar-
rhythmia logbook is shown
in Figure 19.1. A represen-
tative episode is provided
in Figure 19.2.
Figure 19.1  Excerpt from patient’s arrhythmia logbook.
Friedman, Rott, Wokhlu, Asirvatham, Hayes 79

19

Figure 19.2  Representative episode.

Q
:
What intervention is required to address
the ventricular tachycardia episodes?
1. Reposition the RV lead away from the tricuspid annulus
2. Turn on antitachycardia pacing at this heart rate
3. Adjust the atrial sensitivity to correct undersensing
4. Correct lead reversal in the header
80 A Case-Based Approach to Pacemakers, ICDs, and Cardiac Resynchronization

19 4. Correct lead reversal in the header


This is a case of atrial and ventricular lead reversal in the header, result- Intraoperative evaluation confirmed reversal of atrial and ventricu-
ing in inappropriate detection of ventricular tachycardia. Figure 19.2 lar lead connections in the header, and correction of lead connections in
demonstrates that the patient is in atrial flutter by surface ECG. The the header resulted in normal device function.
ventricular EGM demonstrates a very fast rate, 300 bpm, suggestive of In retrospect, the initial postprocedural interrogation strip, shown
fast ventricular tachycardia. However, the atrial EGM corresponds to in Figure 19.3, demonstrated abnormalities that were misinterpreted as
surface QRS and the ventricular EGM, in turn, corresponds to atrial ventricular lead microdislodgment. The absence of a surface EGM hin-
flutter waves. It is interesting that the amplitude of the sensed ventric- dered the immediate recognition of the true problem. The ventricular
ular EGM is so much smaller in comparison to the amplitude of the EGM demonstrated a small sensed R wave (2.7 mV) despite the observa-
sensed atrial EGMs. tion that the R-wave amplitude was normal (> 5 mV) when initially evalu-

Figure 19.3  EGM from initial


postprocedural interrogation.
Friedman, Rott, Wokhlu, Asirvatham, Hayes 81

ated through the pacing system analyzer. As illustrated in Figure 19.3, • Failure to capture the ventricle with VVIR pacing 19
the timing on the A and V channels suggests that intrinsic events on • Failure to mode switch in a patient with known atrial
the “atrial” channel (actually sensed R complexes) intermittently follow arrhythmias
from events on the “ventricular” channel after 90 ms (actually atrial in
nature and consistent with AV conduction). Nevertheless, in the absence The ways to avoid lead reversal during dual-chamber implants
of surface EGMs, the presence of dual-chamber pacing at 60 bpm against include:
the backdrop of intrinsic sinus bradycardia with pauses complicates the • Verify by serial number or atrial lead labeling that leads are
diagnosis of lead reversal. properly inserted. Always connect the ventricular lead first.
Answer 1 is incorrect because moving the lead would not correct • Recheck all parameters when the leads are connected to the
the problem of header reversal. Answer 2 is incorrect because there is device. Dramatic changes in sensing or threshold should
no evidence of failure to pace consistent with an open circuit. Answer 3 catch your attention.
is incorrect because a defibrillator is not indicated. This is atrial flutter, • Ventricular pacing can be tested in VVI mode rather than
and mode switching is the more appropriate device response. Impor- DDI to allow for failure to capture in the event of lead
tantly, however, if this device were a defibrillator, the patient might have reversal. This will minimize the chance that atrial pacing
received inappropriate therapies for ventricular tachycardia. will not be mistaken for ventricular.
• The person who interrogates or a colleague should verify
Signs of lead reversal in the header are as follows: that surface EGMs correspond to the ventricular EGM,
• Surface ECG demonstrates events in the opposite chamber particularly if new, unexplained abnormalities arise.
from what is demonstrated by intracardiac ECGs
• Ventricular EGM amplitude is smaller than expected from
interrogation through the pacemaker system analyzer; atrial
EGM amplitude may be more robust than expected
Case 20
RESYNCHRONIZATION
82 A 53-year-old female with a permanent pacemaker (St. Jude Identity XL DR) recently
PACEMAKERS, ICDs, AND CARDIAC
,

underwent a mechanical tricuspid valve replacement. Due to postoperative cardiac


arrest, her device was reprogrammed from VDDR 60 to 120 bpm to VVI 90 to 120
bpm. Following this, her heart rate repeatedly fell below the lower rate limit without
electrocardiographic evidence of pacing. A representative tracing obtained by
telemetry is shown in Figure 20.1. Despite normal sensing during interrogation, her
ventricular sensitivity threshold was empirically adjusted from 1.5 to 4.0 mV because
of a potential concern for T-wave oversensing.

The next afternoon, you are called to the patient’s bedside because it occurs
again while she is watching television. A 12-lead ECG confirms that the patient has
transitioned to a junctional, nonpaced rhythm. Device interrogation demonstrates
that the ventricular lead has a normal impedance and threshold value, and there is no
evidence of T-wave oversensing. Figure 20.2 demonstrates the heart rate histogram for
the 2 days prior to your assessment.
Friedman, Rott, Wokhlu, Asirvatham, Hayes 83

20
Device settings:
• Mode: VVI
• Pacing rate: 90 to 120 bpm
• V threshold lead: 0.75 V at 0.4 ms
• V impedance: 310 ohms
• Battery voltage: 2.76 V (ERI 2.5 V)
• V sensitivity: 4.0 mV
• Sensor: off
• Rest rate: off
• Maximum sensor rate: 120 bpm
• Hysteresis rate: 50 bpm
• Search interval: off
• AutoCapture: on
84 A Case-Based Approach to Pacemakers, ICDs, and Cardiac Resynchronization

20

Figure 20.1  Surface telemetry.

Figure 20.2  Heart rate histogram.


Friedman, Rott, Wokhlu, Asirvatham, Hayes 85

20

Q
:
What is the likely explanation for the repeated failure to
pace below the lower rate limit?
1. Heart rate hysteresis
2. AV interval hysteresis
3. T-wave oversensing
4. Automated capture algorithm
86 A Case-Based Approach to Pacemakers, ICDs, and Cardiac Resynchronization

20 1. Heart rate hysteresis


There are several potential explanations for paced rates below the pro- In this case, there is failure to output rather than failure to capture
grammed lower rate limit: because no pacing artifact is seen during the slow rates. In addition, the
• Device malfunction due to oversensing by the ventricular loss of pacing occurs with a brisk transition to a much slower, intrinsic
lead (due to extracardiac interference or inappropriate rhythm. The histogram confirms that there is no ventricular pacing be-
sensing of other cardiac events, eg, T-wave oversensing) low the lower rate limit—only sensing. This transition occurred repeat-
• Open circuit edly at various times of the day when the patient was awake but not
• Battery depletion active, as stated in this example. This scenario is most consistent with
• Automated capture algorithms heart rate hysteresis.
• PVARP extension algorithms after frequent premature Figure 20.3 illustrates how heart rate hysteresis works. Retrospec-
ventricular contractions tive evaluation of the strips suggested that there was never clear evidence
• Rest or sleep mode of T-wave oversensing. Hysteresis was inadvertently turned on when pro-
• Hysteresis gramming changes were made after the cardiac arrest. Figure 20.4 dem-

Figure 20.3  Ventricular


hysteresis in VVIR mode.
Friedman, Rott, Wokhlu, Asirvatham, Hayes 87

onstrates improvement of the heart rate histogram after programming incorrect because there is a sensing of a premature ventricular complex, 20
hysteresis to “off.” rather than T-wave oversensing. This results in inhibition of output and
Answer 2 is incorrect because the patient is in a VVI mode with initiates the hysteresis escape interval. Answer 4 is incorrect because
a silent atrium that is not being sensed. Search positive AV interval hys- automated capture algorithms usually work in concert with a backup
teresis is a specific type of hysteresis in which there is periodic prolon- safety pulse when capture fails. A marked and lasting change to a slow
gation of the PR interval after a ventricularly paced event. Answer 3 is intrinsic rate would be inconsistent with automated capture algorithms.

Figure 20.4  Two days later, after programming hysteresis to “off.”


88 A Case-Based Approach to Pacemakers, ICDs, and Cardiac Resynchronization

20 In addition to identifying hysteresis as the source of the problem


of low rates, you identify another problem while interrogating the real-
time EGMs related to the recent adjustment of ventricular sensitivity, as
shown in Figure 20.5.

Figure 20.5  Real-time EGM after increase in ventricular sensitivity level.


Friedman, Rott, Wokhlu, Asirvatham, Hayes 89

20

Q
:
What is the consequence of empirically changing the
ventricular sensitivity level from 1.5 to 4.0 mV?
1. Higher likelihood of T-wave oversensing
2. Atrial oversensing
3. Ventricular undersensing
4. Inappropriate activation of hysteresis
90 A Case-Based Approach to Pacemakers, ICDs, and Cardiac Resynchronization

20 3. Ventricular undersensing
Making the ventricular sensing channel less sensitive may be beneficial A potentially lethal consequence of ventricular undersensing is pacing
to limit the potential for sensing noise and double counting of T waves, on the T wave, which can induce ventricular tachyarrhythmias. R on T
but in this case, where the R wave is small, reprogramming resulted in resulting in a ventricular tachyarrhythmia is more likely to occur in the
ventricular undersensing. Figure 20.6 illustrates how this occurs. First, patient with significant LV dysfunction.
there is failure to sense the intrinsic ventricular event (R), followed by a Answer 1 is incorrect because raising the sensitivity level reduces
pacing artifact, which at times occurs coincident with the T wave. Pacing the risk of T-wave oversensing. Answer 2 is incorrect because raising the
artifacts without capture represent functional failure to capture when sensitivity level reduces the risk of sensing far-field atrial events. Answer
the ventricle is refractory. When the pacing is attempted once sufficient 4 is incorrect because undersensing predisposes to overpacing in the
time elapses after an intrinsic ventricular event, ie, when the ventricular ventricle. Ventricular hysteresis is usually triggered by situations of in-
myocardium is no longer refractory, ventricular depolarization occurs. trinsic ventricular activity.
Friedman, Rott, Wokhlu, Asirvatham, Hayes 91

20

Figure 20.6  Ventricular undersensing; sensitivity threshold, 0.4 mV.


Case 21
RESYNCHRONIZATION
92 Figure 21.1  Medtronic Cardiac
PACEMAKERS, ICDs, AND CARDIAC
,

Compass report.
Friedman, Rott, Wokhlu, Asirvatham, Hayes 93

21
A 71-year-old female was implanted with a dual-chamber ICD 6 years ago due to
inducible monomorphic ventricular tachycardia and ischemic cardiomyopathy. A
routine remote transmission reveals the Medtronic Cardiac Compass report shown in
Figure 21.1.

Q
:
On the basis of this report, what events occurred
in late May 2010?
1. Multiple shocks
2. Increase in patient activity
3. AT/AF episodes with elevated ventricular rate
4. Increase in nonsustained episodes
94 A Case-Based Approach to Pacemakers, ICDs, and Cardiac Resynchronization

21 3. AT/AF episodes with elevated ventricular rate


The report shows occurrence of AT/AF at the end of May 2010. During line percentage ventricular pacing over the next 3 months. This sudden
these episodes of AT/AF, the ventricular rate was elevated, averaging major change in AV nodal conduction is most likely due to the effect of
about 100 bpm with maximum ventricular rates at 150 to 200 bpm. amiodarone. Amiodarone was taken for only 1 month, so the amount of
Answers 1 and 4 are incorrect, because there were no treated VT/ ventricular pacing decreased as the amiodarone effect slowly dissipated
VF episodes or nonsustained VT occurring during the entire time frame over 4 to 8 weeks after discontinuation of the drug.
encompassed in this report. Answer 2 is incorrect, since the patient’s In cases where the actual clinical history is not known, the Cardiac
activity actually abruptly decreased in late May 2010, and then gradually Compass can reliably provide similar information to the medical record.
increased back to the baseline activity level of 3 h/day. Consider the following device programmed parameters in order to
Further investigation of the patient’s medical records reveals she answer the question on the opposite page.
underwent repeat sternotomy for a mitral valve replacement and coro-
nary artery bypass grafting on May 18, 2010. Immediately postop, she Preop brady parameters:
experienced atrial fibrillation with rapid ventricular response resulting • Mode: DDDR
in the initiation of amiodarone for 1 month and coumadin for 6 weeks. • Pacing rate: 60 to 120 bpm
The patient’s sudden decrease in activity reflects the recovery pe- • Paced and sensed AV delays: 280 ms
riod immediately following cardiac surgery, and the steady increase in
activity with the assistance of cardiac rehab over the next month and a Postop brady parameters:
half. • Mode: DDD
The ventricular pacing rate also increased both during the AT/ • Pacing rate: 60 to 120 bpm
AF episodes and immediately following, gradually decreasing to base- • Paced and sensed AV delays: 280 ms
Friedman, Rott, Wokhlu, Asirvatham, Hayes 95

21

What is the most likely cause for both the day and night

Q
:
average ventricular rate remaining at 60 bpm from the
end of May 2010 to early September 2010?
1. Postoperative reprogramming failed to turn rate response on
2. Frequent ventricular arrhythmia
3. Device in persistent mode switch due to AT/AF
4. Magnet response
96 A Case-Based Approach to Pacemakers, ICDs, and Cardiac Resynchronization

21 1. Postoperative reprogramming failed


to turn rate response on report shows the daytime average ventricular rate slightly increased
Review of the pre- and postop programming revealed that rate response again.
was programmed on preop but was not programmed back on postop. Answer 2 is incorrect because the report shows neither treated nor
Cardiac rehab staff members noted that the patient’s heart rate was nonsustained ventricular arrhythmia occurred throughout the entire
always at 60 bpm throughout her entire exercise, whether it be on the time frame of the report. Answer 3 is incorrect, for the report shows
treadmill, exercise bike, rowing machine, or elliptical machine and for no further episodes of AT/AF after the occurrences in late May 2010.
any duration of activity. Arrangements were scheduled to have the rate Answer 4 is incorrect, for ICDs respond to magnet application only by
response programmed back on in early September, at which time the inhibiting tachy detection, not by changing pacing parameters.
Friedman, Rott, Wokhlu, Asirvatham, Hayes 97

21
Case 22
RESYNCHRONIZATION
98 A 77-year-old male sends an unscheduled remote transmission, as he believes
PACEMAKERS, ICDs, AND CARDIAC
,

he received a shock from his ICD while asleep. He first received a dual-chamber
pacemaker 11 years ago due to atrial fibrillation status post AV node ablation. Four
years later, the pacemaker was upgraded to a CRT-P device due to nonischemic
dilated cardiomyopathy, and then 1 year later the device was upgraded to a CRT-D
device due to ventricular dysfunction and heart failure. Over the past 4 years, he
developed worsening heart failure and ventricular tachycardia requiring ICD shocks,
necessitating medication adjustments and hospitalizations. In addition, his quality of
life diminished due to difficulty participating in his favorite pastimes of hunting and
fishing.
Friedman, Rott, Wokhlu, Asirvatham, Hayes 99

22

Figure 22.1  Cardiac Compass OptiVol fluid index.


100 A Case-Based Approach to Pacemakers, ICDs, and Cardiac Resynchronization

22
Figure 22.2  Cardiac Compass report.
Friedman, Rott, Wokhlu, Asirvatham, Hayes 101

22

Q
:
On the basis of the reports outlined in Figures 22.1 and
22.2, what happened to the patient in early June 2010?
1. Increase in percent pacing
2. Sudden increase in fluid volume
3. Increase in patient activity
4. Termination of AT/AF
102 A Case-Based Approach to Pacemakers, ICDs, and Cardiac Resynchronization

22 2. Sudden increase in fluid volume


The report shows sudden changes in the OptiVol fluid index, thoracic The sudden decrease in impedance is consistent with pulmonary
impedance, and ventricular rate during AT/AF, average ventricular rate edema. Noteworthy is that this occurred at the time of LVAD implanta-
both day and night, and patient activity hours per day. Review of the tion, and additional causes for the change of impedance may include the
medical records show implantation of a left ventricular assist device on contribution to lowering impedance by the metal of the LVAD itself and
June 8, 2010. (Please refer to Figure 22.3, which shows the last 90-day RV-LV function mismatch.
zoom of this report.) The average ventricular rate during AT/AF and during the day
Answer 1 is incorrect as the % Pacing/day, usually near 90%, actu- and night increased following LVAD implant because the pacing lower
ally decreases to about 50% in early June. Answer 3 is incorrect, for the rate limit was reprogrammed to 90 bpm immediately postop, and then
patient activity actually decreases from 1 h/day to 0 h/day. Answer 4 is decreased to 80 bpm about a month and a half later. The surgical team
incorrect, for the AT/AF trend continues as usual in early June. requested the increase in lower rate limit function, as it appeared to
In mid-May 2010, OptiVol fluid index and thoracic impedance benefit LVAD function.
suggest an increase in fluid volume, as the thoracic impedance drops The patient activity hours per day suddenly decreased from 1 h/
slightly below the reference line. At this time, it was decided to implant day to 0 h/day due to the hospitalization and recovery period following
an LVAD device due to worsening heart failure. The daily impedance LVAD implant. His activity continues to be diminished even 2 months
then returns to the reference line, and then in early June, abruptly from LVAD implant.
changes following the LVAD implant.
Friedman, Rott, Wokhlu, Asirvatham, Hayes 103

22

Figure 22.3  Cardiac Compass OptiVol fluid index—last 90-day zoom.


Case 23
RESYNCHRONIZATION
104 An 89-year-old male received a CRT-D device 1 year ago. He has history of coronary
PACEMAKERS, ICDs, AND CARDIAC
,

artery disease, congestive heart failure, aortic valve replacement, and chronic atrial
fibrillation. He has received appropriate shocks for presyncopal tachyarrhythmia.

Device settings:
• Tachy VF zone: 188 bpm
• Therapies: ATPx1 during charging; 35-J shock x 6
• Brady mode: VVIR
• Pacing rate: 60 to 120 bpm
• Mode switch: on.
• RV lead parameters: output 2.5 V at 0.4 ms; sensitivity 0.3 mV
• LV lead parameters: output 2.5 V at 0.4 ms; pace polarity: LV tip to LV ring
Friedman, Rott, Wokhlu, Asirvatham, Hayes 105

23
Figure 23.1  Ventricular
fibrillation episode EGM.

Q
:
Why did the device deliver a shock after ATP slowed
the ventricular rhythm (Figure 23.1)?
1. Oversensing of ventricular rhythm
2. Far-field sensing of atrial arrhythmia
3. All shocks are committed once charging begins
4. Ventricular fibrillation resumed just prior to the charge ending
106 A Case-Based Approach to Pacemakers, ICDs, and Cardiac Resynchronization

23 4. Ventricular fibrillation resumed


just prior to the charge ending Answers 1 and 2 are incorrect, for the ventricular markers match
The device appropriately detected the ventricular tachycardia (VT), consistently with each R-wave deflection on the ventricular EGM, with
which started during an ongoing atrial tachyarrhythmia. ATP during no oversensing of other signals; specifically, the ongoing atrial signals
charging successfully terminated the VT, with resumption of biventricu- are not oversensed. If an ICD delivers a shock following an initial de-
lar pacing. Just before the charge ended (“CE” marker), the VT recurred. tection irrespective of whether the tachycardia terminates before shock
Thus, the device appropriately reconfirmed VT during the reconfirma- delivery, the shock is said to be “committed.” Answer 3 is incorrect, for
tion that occurs immediately following charge end and a shock was deliv- the first shock is never committed; reconfirmation always occurs. In this
ered, slowing the ventricular rate below the VF detection rate. case, tachycardia recurred during reconfirmation, leading to therapy.
Note how the episode plot in Figure 23.2 shows that the VV inter-
vals return to near 600 ms following the burst ATP, then just before the
34.6-J shock the cycle length shortens (heart rate increase) to 200 ms.
Friedman, Rott, Wokhlu, Asirvatham, Hayes 107

23

Figure 23.2  Episode plot.


Case 24
RESYNCHRONIZATION
108 A 22-year-old female with a history of ischemic cardiomyopathy, myocardial infarction,
PACEMAKERS, ICDs, AND CARDIAC
,

LV aneurysm, and polysubstance abuse received a dual-chamber ICD for secondary


prevention of sustained monomorphic ventricular tachycardia. She sent a remote
Latitude transmission after receiving multiple consecutive shocks that occurred during
emotional stress (Figure 24.1).

Device settings:
• VF zone: 185 bpm
• Therapies: one sequence of ATP followed by eight 41-J shocks
• VT zone: 150 bpm
• Therapies: 3 sequences of ATP followed by six 41-J shocks
• VT zone detection enhancements: on
• Initial: V rate > A rate
• AFib rate threshold: 170 bpm
• Stability: 20 ms AND Onset 9%
• Sustained rate duration: 3 minutes
Friedman, Rott, Wokhlu, Asirvatham, Hayes 109

• Postshock: V rate > A rate


• AFib rate threshold: 170 bpm
24
• Stability: 20 ms
• Sustained rate duration: 15 seconds
• Mode: DDD
• Pacing rate: 50 bpm (lower rate), 130 bpm (upper rate)
• Mode switch: on
• AV delays: 180 ms (paced), 150 ms (sensed), search AV on

Figure 24.1  Initial episode EGM.


110 A Case-Based Approach to Pacemakers, ICDs, and Cardiac Resynchronization

24

Figure 24.2  Continuation of EGM.


Friedman, Rott, Wokhlu, Asirvatham, Hayes 111

24

Q
: Why was therapy delivered?
1. Rhythm monomorphic VT
2. Episode fell into VF zone with no detection enhancements
3. Sustained rate duration timed out
4. ATP treating atrial tachycardia
112 A Case-Based Approach to Pacemakers, ICDs, and Cardiac Resynchronization

24 3. Sustained rate duration timed out


Figure 24.1 shows the initial episode EGM with 1:1 AV conduction with crease sensitivity but decrease therapy specificity by overriding SVT-VT
the heart rate gradually (note Gradl marker) increasing into the VT zone discriminators to force therapy delivery. In this example, the timer led
at 170 bpm (353 ms). Detection enhancements of V>A Rate, and AFib to an inappropriate shock.
are listed as “False” on the episode report (Figure 24.3). False indicates Answer 1 is incorrect, since EGMs show 1:1 AV conduction with
VT is not present, so therapy is withheld. Note the “True” after “SRD,” a gradual onset, indicating regular sinus tachycardia. Answer 2 is incor-
or sustained rate duration. SRD is also seen on Figure 24.2 just above rect since the markers indicate that the rates fell into the VT zone, as
“V-Detect.” The sustained rate duration times out at the programmed 3 they are labeled VT and the intervals do not reach the programmed VF
minutes, so even though the other detection enhancements do not indi- zone of 200 bpm (300 ms). Answer 4 is incorrect since the EGMs do not
cate VT, therapy is delivered. Sustained duration timers are used under show the delivery of atrial, but rather the delivery of ventricular, ATP.
the assumption that a sustained tachycardia is more likely VT; they in- Indeed, this device does not have the capability of delivering atrial ATP.
Friedman, Rott, Wokhlu, Asirvatham, Hayes 113

24

Figure 24.3  Episode detail.


Case 25
RESYNCHRONIZATION
114 A routine transtelephonic tracing was received from an 89-year-old female. The
PACEMAKERS, ICDs, AND CARDIAC
,

original pacemaker was placed 10 years earlier, and over the years the patient had
become increasingly pacemaker-dependent and rarely displayed intrinsic rhythm
on the transtelephonic transmission. At the time of the transmission (Figure 25.1),
the patient was residing in a nursing home and assisted by a nursing aide. Vitals
immediately prior to the transmission documented a pulse rate of 65 bpm.

Device settings:
• Mode: VVIR
• Pacing rate: 65 to 130 bpm
• Amplitude: 2.5 V
• Pulse width: 0.5 ms
• Sensitivity: 2.8 mV
• VRP: 330 ms
Friedman, Rott, Wokhlu, Asirvatham, Hayes 115

25

Figure 25.1  Nonmagnet tracing.

Q
:
Given the apparent discrepancy with past transtelephonic
transmissions, what would be the most logical next step?
1. Have patient schedule an in-clinic visit to evaluate and reprogram sensitivity
2. Discuss with the nursing home aide and then repeat the transmission
3. Unless elective replacement indicators are present, ignore the transmission
4. Immediate ER assessment to avoid R-on-T-induced tachyarrhythmia
116 A Case-Based Approach to Pacemakers, ICDs, and Cardiac Resynchronization

25 2. Discuss with the nursing home aide and then


repeat the transmission bpm, is no longer present.
An annotated version of the transmission is shown in Figure 25.2. The In close questioning of the caregiver helping the patient with the
tracing suggests failure to sense the intrinsic rhythm and uncertainty transmission, the caregiver had been in physical contact with the trans-
regarding capture. However, as noted previously, the patient had not telephonic bracelets during the first transmission. This allowed the care-
displayed an intrinsic rhythm on transtelephonic transmission for a long giver’s intrinsic rhythm to be recorded on the transmission. When the
time. transmission was repeated without the caregiver in contact, the trans-
The nurse asked the caregiver to repeat the transmission and ques- mission was straightforward and revealed normal function in keeping
tioned the caregiver carefully about how the transmission was being ac- with the patient’s past transmissions.
complished. A subsequent nonmagnet transmission is shown in Figure Of other potential choices in the original question, one should
25.3. never ignore an electrocardiographic finding simply because it was not
This transmission demonstrates complete pacing with ventricular understood and “probably” benign, ie, answer 3. From a practical/
capture at a rate of approximately 65 bpm (Figure 25.3). (The rate is logistical standpoint it would also be ideal to avoid sending an elderly
probably slightly above the programmed lower rate limit as a result of patient to the emergency department unnecessarily (answer 4). Bringing
sensor-driven pacing related to the activity incurred during the transmis- the patient into the pacemaker clinic, answer 1, would not be unreason-
sion.) The intrinsic rhythm, previously at a rate of approximately 100 able but the nurse or technician with significant transtelephonic experi-
ence would recognize the possible abnormality based on the tracing.

Figure 25.2  Annotated version of transmission shown in Figure 25.1


Friedman, Rott, Wokhlu, Asirvatham, Hayes 117

25

Figure 25.3  Repeat transtelephonic transmission.


Case 26
RESYNCHRONIZATION
118 A 68-year-old man receives a permanent pacemaker 3 months earlier for intermittent
PACEMAKERS, ICDs, AND CARDIAC
,

AV block. He was minimally symptomatic prior to the pacemaker. He returns for a


routine 3-month follow-up.

Device settings:
• Mode: DDD
• Pacing rate: 60 to 110 bpm
• PAV delay: 175 ms
• SAV delay: 150 ms
• V output: 0.4 ms at 4.0 V
• A output: 0.4 ms at 4.0 V
• AV hysteresis options: off

The 12-lead EGM without magnet application shown in Figure 26.1 was obtained. Next,
a magnet is placed to force DOO pacing, and an additional tracing is obtained, shown
in Figure 26.2.

Nonmagnet and magnet tracings reveal atrial pacing with an AR interval of


approximately 280 ms, ie, significantly longer than the programmed PAV delay
(175 ms).
Friedman, Rott, Wokhlu, Asirvatham, Hayes 119

26

Figure 26.2  Magnet application.

Figure 26.1  Nonmagnet 12-lead EGM.

Q :
Based on your EGM diagnosis and the programmed
parameters, what could explain these findings?
1. Ventricular lead dislodgment
2. Connector pin not fully inserted in the header
3. Crosstalk in the absence of safety pacing
4. Ventricular avoidance pacing algorithm
120 A Case-Based Approach to Pacemakers, ICDs, and Cardiac Resynchronization

26 2. Connector pin not fully inserted in the header


The differential diagnosis for the finding of “ventricular failure to out- Additional information from the interrogation includes a ven-
put” should be considered. The ventricular output could be inhibited tricular impedance of > 1990 ohms. For this particular pulse generator
because some other cardiac or extraneous activity is being sensed con- this value represents infinity and is consistent with an open circuit. It
sistently. Alternatively, the electrical circuit may be open, preventing the does not differentiate between a connector pin that is not secured in the
pacemaker output from reaching the heart. The most likely causes of header and a fracture of the conductor coil.
an open circuit would include a fracture of the conductor coil or the The EGM with markers is shown in Figure 26.3. The markers
connector pin not being secured adequately in the header of the pulse indicate that both atrial and ventricular outputs are being delivered,
generator. consistent with the programmed parameters. However, the ventricular

Figure 26.3  EGM with markers.


Friedman, Rott, Wokhlu, Asirvatham, Hayes 121

pacing artifact is not apparent on the corresponding surface EGM and Answer 1 is incorrect because with lead dislodgment there are still 26
the actual AR interval is significantly greater than the programmed AV ventricular pacing artifacts present and the presentation is failure to
interval. capture, not failure to output. Answer 3 is incorrect because crosstalk,
A chest x-ray was obtained (Figure 26.4). The close-up of the pulse or oversensing of atrial signal on the ventricular lead, results in failure
generator is seen, and on inspection of the header of the device, the to output in nonmagnet mode, but not in a nonsensing magnet mode.
ventricular lead is not completely inserted into the header. This is most Forced ventricular pacing artifacts would still be present. Similarly, an-
evident when the tips of the atrial and ventricular connector pins are swer 4 is incorrect because ventricular avoidance algorithms rely on sens-
compared. An invasive approach to secure the lead in the connector ing and, therefore, are not operative during magnet mode.
block is required in this situation.

Figure 26.4  Patient’s chest x-ray.


Case 27
RESYNCHRONIZATION
122 A 72-year-old female undergoes a dual-chamber pacemaker implant for sinus node
PACEMAKERS, ICDs, AND CARDIAC
,

dysfunction. The pacemaker was programmed to a DDDR pacing mode at the end of
the procedure with AV interval of 220 ms. Thresholds obtained during the implant
via the pacing systems analyzer with cables connected directly to the leads were
excellent: A = 0.5 ms and 0.7 V, impedance 574 ohms; V = 0.5 ms and 0.5 V, impedance
690 ohms.

Within an hour of the patient returning to her hospital room, the pacemaker service
was called because of the tracing obtained on telemetry shown in Figure 27.1.
Friedman, Rott, Wokhlu, Asirvatham, Hayes 123

27

Figure 27.1  Telemetry tracing.

Q
: What would be your next step?
1. Activate the PMT algorithm
2. Turn off “sudden-brady” response
3. Invasively revise the pacing system
4. Shorten the AV interval
124 A Case-Based Approach to Pacemakers, ICDs, and Cardiac Resynchronization

27 3. Invasively revise the pacing system


Without more information the tracing is very difficult to interpret. Fig- mately 220 ms, ie, the programmed AV interval. This suggests that the
ure 27.2 shows the initial tracing with annotations. intrinsic V is being seen on the atrial channel and initiating the AV
Depending on where one attempts to initiate the tracing, it can be interval and a subsequent pacing artifact, although it is actually coming
approached as a pacing artifact followed by a native QRS at a relatively from the atrial lead.
regular “artifact to QRS” interval, or a native QRS interval followed by The EGM with markers makes the interpretation much simpler
a pacing artifact at a relatively regular interval. The “artifact to QRS” (Figure 27.3).
interval is approximately 280 to 300 ms. The “QRS to artifact” inter- Comparing the surface EGM to the atrial and ventricular EGM
val, depending on where the intrinsic deflection of the QRS is actually and markers, it is clear that the intrinsic atrial events are marked as
sensed, is therefore more difficult to measure but appears to be approxi- VS, or ventricular sensed, events and the intrinsic ventricular events

Figure 27.2  Annotated tracing.


Friedman, Rott, Wokhlu, Asirvatham, Hayes 125

are seen as AS, or atrial sensed, events. The ( ) indicate that all the AS equal to or near the maximum tracking rate. A Sudden Brady Response 27
events occur in a refractory period. This is consistent with lead reversal, (answer 2) is incorrect because one would observe a sudden increase in
ie, the atrial lead has been plugged in the ventricular port and vice versa. paced rate (AP-VP), which is not present. Shortening the AV interval
System revision was required. (answer 4) might seem to be a possibility when reviewing the original
There are no noninvasive options that would correct this prob- tracing without markers, but the markers clearly identify a lead reversal,
lem. Activating the PMT algorithm (answer 1) assumes the problem is so that answer 4 is incorrect.
that of PMT. With PMT, the markers would demonstrate AS-VP at rates

Figure 27.3  EGM with markers.


Case 28
RESYNCHRONIZATION
126 An asymptomatic 76-year-old female is seen for routine follow-up. She is 9 months
PACEMAKERS, ICDs, AND CARDIAC
,

postimplant and has been followed elsewhere.

Device settings:
• Mode: AAIR
• Pacing rate: 75 to 160 bpm
• Output: 2.2 V at 0.5 ms
• Sensitivity: 1 mV
• Refractory: 280 ms
• VRR: on
• Bipolar
• Atrial threshold: 1.4 V at 0.5 ms
Interrogation reveals the arrhythmia
logbook shown in Figure 28.1.

The tracing in Figure 28.2 represents the


predominant rhythm seen during follow-up.

Figure 28.1  Patient’s arrhythmia logbook.


Friedman, Rott, Wokhlu, Asirvatham, Hayes 127

28

Figure 28.2  Patient’s tracing.

How do you put together the findings of frequent events

Q
:
labeled as tachyarrhythmias, far-field sensing, and no
alteration in the pacing rate?
1. Far-field events have occurred during the atrial refractory period
2. Tachycardia events are resetting timing cycle
3. Tachycardias are real
4. Initial atrial pace fails to capture and a safety pace is delivered in refractory
128 A Case-Based Approach to Pacemakers, ICDs, and Cardiac Resynchronization

28 1. Far-field events have occurred during the atrial


refractory period rhythmia logbook, ie, the tachycardias are not real (answer 3).
When the tracings are reviewed it is critical to understand the markers Despite the arrhythmia logbook being potentially misleading, the
used (Figure 28.3). Figure 28.3 shows the surface ECG (top) and intra- far-field sensing is not resulting in any alterations in timing or therapy
cardiac electrogram (bottom). Note that a large intracardiac deflection delivery (answer 2). In addition, although the atrial high-rate episodes
occurs before the surface QRS, consistent with atrial activity, and that are at different cycle lengths, the onset and maximum heart rate are
the smaller electrogram represent far-field sensing of the R-wave. Below extremely similar for each recorded episode. This should serve as a clue
the tracings are the keys to the markers used in the tracing of Figure that this is oversensing. Note that the tachycardias in the logbook have
28.2. In the example in Figure 28.2, “P-Sr” indicates sensor-driven atrial a rate of around 300 bpm. That rate corresponds to a cycle length of
pacing compatible with the programmed pacing mode. The (S) that 200 ms—the time interval between the paced complex and the refractory
follows each “P-Sr” indicates a sensed event that occurs in a refractory sense (S) event. When the cycle length exceeds the programmed upper
period (the parenthesis around the S indicate that it occurred during a rate interval, eg, this device is programmed to an upper rate limit of 160
refractory period). bpm (375 ms), the rate is classified as a “tachycardia.” When a cycle oc-
Far-field sensing of the native QRS complex occurs but does not curs without an (S) event, the cycle length falls below the programmed
alter pacing timing because it occurs in the refractory period. Howev- upper rate interval and the tachycardia is considered terminated.
er, sensed events that occur during refractory periods are counted for Answer 4 could not be correct because there is no failure to cap-
purposes of arrhythmia detection. The consistent “double-counting” ture noted and ‘safety pacing’ would not be a feature when programmed
satisfies criteria for a “tachycardia” and explains the findings on the ar- to a single-chamber pacing mode.
Friedman, Rott, Wokhlu, Asirvatham, Hayes 129

28

Figure 28.3  Patient tracing (top: surface ECG, bottom: intracardiac electrogram) and key to markers.
Case 29
RESYNCHRONIZATION
130 A 73-year-old male returned for routine follow-up of a DDD device. Although he had
PACEMAKERS, ICDs, AND CARDIAC
,

not previously mentioned any symptoms during prior transtelephonic transmissions, at


the time of the routine visit he noted episodes of light-headedness and subsequently
feeling as though his pulse was going too fast. He also noted a very vague “shocking”
sensation in his left arm that seemed to coincide with the fast heart rate.

Device settings: Measured data:


• Mode: DDD • P wave: 2 mV; R wave > 11 mV
• Mode switch: off • Thresholds: A = 1 V at 0.6 ms; V = 1 V
• LR: 60; UR: 120 at 0.8 ms
• AVI: 240/210; RRAVD: off
• Amplitude: A = 5.0; V = 2.5 Interrogation also notes frequent
• Pulse width: A = 0.6; V = 0.45 Rate-Drop Response (RDR) episodes.
• Sensitivity: A = 0.25 mV; No stored EGMs are available. The
V = 2.8 mV nonmagnet tracing is shown in Figure
• Rate-Drop Response on: top 90; 29.1, and another tracing seen in clinic
bottom 70; confirmation beats 5 is shown in Figure 29.2.
Friedman, Rott, Wokhlu, Asirvatham, Hayes 131

29

Figure 29.2  In clinic observation. From top to bottom


are shown: surface ECG, markers, and atrial electrogram.
Figure 29.1  Nonmagnet tracing. From top to bottom are shown:
surface ECG, markers, and atrial electrogram.

Q : What is the most likely diagnosis?


1. Intermittent failure to capture
2. Inappropriate response from rate-adaptive sensor
3. Pacemaker syndrome
4. Atrial or ventricular tachyarrhythmias
132 A Case-Based Approach to Pacemakers, ICDs, and Cardiac Resynchronization

29 3. Pacemaker syndrome
Rate drop (RDR) is a programmable feature that detects a predefined feeling light-headed, attributable to the pacemaker syndrome, followed
dramatic drop in heart rate and results in pacing at a significantly faster by the fast heart rate, attributable to the RDR.
rate for a defined period. The algorithm is used in patients with neu- During this episode the patient spontaneously noted that he was
rocardiogenic syncope with significant cardioinhibition. Rapid pacing experiencing the symptoms previously described, ie, racing heart and
may help to minimize the blood pressure fall that often accompanies the mild light-headedness.
cardioinhibitory response. (RDR is a trademark of Medtronic. Several No definite evidence suggests that there may be a loss of integrity
manufacturers have variations of this algorithm, eg, the Sudden Brady on the atrial lead, ie, repetitive impedance measurements were normal
Response [SBR] of Boston Scientific). and there is nothing to support a diagnosis or lead insulation on conduc-
In this case, without the benefit of EGMs for review an actual tor coil abnormality.
RDR episode, it is impossible to know if the RDR episodes were respon- The options would be to turn off the RDR algorithm or to ad-
sible for his symptoms and whether or not they were appropriate, ie, just the sensitivity on the atrial lead, ie, less sensitive, retest with the
were they initiated by a bradyarrhythmia that met RDR criteria. isometric maneuvers, and see if detection of noise could be avoided. In
The nurse specialist seeing the patient documented acceptable this patient it was possible to alter the atrial sensitivity to avoid noise
atrial and ventricular pacing thresholds with adequately programmed detection but still have adequate sensing of intrinsic atrial events. Once
pacing margin of safety (answer 1) and also performed isometric maneu- this was accomplished the patient had no further symptomatic episodes.
vers, and the tracing in Figure 29.3 was obtained. No spontaneous atrial or ventricular tachyarrhythmias had been docu-
Note that the noise artifact that occurred during isometrics result- mented (answer 4).
ed in the pacemaker falsely recognizing noise on the atrial lead as atrial The patient’s symptoms of palpitations could have been explained
activity, which was tracked, resulting in a rate that is significantly higher by appropriate RDR and awareness of faster pacing or native tachyar-
than the patient’s actual intrinsic rate. At the end of isometrics, noise rhythmias. Options would include turning the RDR off or reprogram-
artifact ceases and there is a relative bradycardia. This change in rate was ming the RDR criteria and observation.
enough to meet the RDR criteria, so that the pacing rate was increased, As noted in the initial description, the pacing mode was DDD;
leading to symptoms. This sequence is consistent with his symptoms of therefore, inappropriate rate-adaptive sensor response would be incor-
rect (answer 2).
Friedman, Rott, Wokhlu, Asirvatham, Hayes 133

29

Figure 29.3  Tracing after isometric maneuvers. From top to


bottom are shown: surface ECG, markers, and atrial electrogram.
Case 30
RESYNCHRONIZATION
134 A 15-year-old with Ebstein’s anomaly and severe tricuspid regurgitation presented
PACEMAKERS, ICDs, AND CARDIAC
,

with complete heart block following tricuspid valve replacement. A dual-chamber


pacemaker was placed.

Device settings:
• Mode: DDD
• Pacing rate: 60 to 170 bpm (185 is maximum programmable tracking
and sensor rate)
• PVARP: 250 ms
• Paced AVI = 170 ms; sensed AVI = 140 ms
• V output: 5.0 V and 0.5 ms
• A output: 5.0 V and 0.4 ms

Four months later the patient’s local cardiologist reported that the patient, who
was very active and a competitive tennis player, would develop sudden and marked
fatigue during heavy exertion. The cardiologist had reprogrammed the pacemaker to
a maximum tracking rate of 185 bpm, the highest available on the device, but the
patient remained symptomatic.
Friedman, Rott, Wokhlu, Asirvatham, Hayes 135

The patient returned and a treadmill exercise assessment was performed. The patient 30
suddenly noted his typical exercise limiting symptoms at 9 minutes, 32 seconds and
exercised to maximum duration of 9 minutes, 47 seconds. The peak exercise tracing is
shown in Figure 30.1.

Figure 30.1 
Patient’s tracing
at peak exercise.

Q
:
Given the patient’s quality of life impairment and the
exercise ECG findings, what is your next step?
1. Program the rate-adaptive sensor to a more sensitive threshold setting
2. Replace the device with one that has a higher programmable maximum tracking rate
3. Shorten the AV interval and PVARP to yield shorter TARP
4. Restrict patient’s activity level
136 A Case-Based Approach to Pacemakers, ICDs, and Cardiac Resynchronization

30 2. Replace the device with one that has a higher


programmable maximum tracking rate Consideration could be given to programming the rate-adaptive
The tracing demonstrates pseudo-Wenckebach behavior when the up- sensor to a more sensitive threshold in an effort to achieve sensor-driven
per rate limit is reached. The patient became symptomatic with the first rate-smoothing at peak rates. In order to achieve less cycle length varia-
occurrence of pseudo-Wenckebach behavior and had to terminate the tion with rate-adaptive pacing for this patient, and for many patients
exercise. Had the patient continued to exercise, as the intrinsic atrial who are very fit and able to increase their intrinsic atrial rate to very
rate had continued to increase, 2:1 upper rate behavior would have been high rates, the sensor would have to be programmed to such a sensitive
seen. level that the sensor would dominate the intrinsic rhythm even when the
patient was at minimal activity levels (answer 1).
Friedman, Rott, Wokhlu, Asirvatham, Hayes 137

Shortening the TARP, answer 3, would not make a difference even The only viable option for this patient was to replace the device 30
if the programming option exists because the maximum achievable track- with one capable of achieving higher maximum tracking rates. The new
ing rate in this device is 185 bpm. pulse generator was programmed to its maximum programmable value
In a patient who is physically active and able to achieve high ven- of 200 bpm. At this setting the patient was able to complete competitive
tricular rates that are appropriate for the level of activity, and in whom tennis games without becoming symptomatic.
there is no contraindication for strenuous activity, eg, a patient with hy-
pertrophic cardiomyopathy and a history of sudden death, it is difficult
to justify or enforce lifelong limitation of a desired activity (answer 4).
Case 31
RESYNCHRONIZATION
138 An elderly male receives a dual-chamber pacemaker for intermittent AV block.
PACEMAKERS, ICDs, AND CARDIAC
,

The tracings shown in Figure 31.1 were obtained the day following the pacemaker
implantation.

The nonmagnet tracing demonstrates P-synchronous pacing, ie, ventricular capture


and atrial sensing are verified. The magnet tracing reveals DOO pacing with the
magnet rate being competitive with the native sinus rate of approximately 90 bpm.
However, there appears to be definite atrial capture with the third, fourth, and fifth
atrial pacing artifacts.

The patient returns at 3 months for a routine evaluation. He has remained


asymptomatic. Only the magnet tracing shown in Figure 31.2 is available.
Friedman, Rott, Wokhlu, Asirvatham, Hayes 139

31

Figure 31.1  Patient tracings, day 1 (upper tracing without Figure 31.2  Three-month follow-up magnet tracing.
magnet; lower tracing with magnet application).

Q : What is the abnormality in the 3-month tracing due to?


1. Loose set screw on atrial lead
2. Atrial lead dislodgment
3. Reversal of leads in connector block
4. Atrial lead conductor coil fracture
140 A Case-Based Approach to Pacemakers, ICDs, and Cardiac Resynchronization

31 2. Atrial lead dislodgment


At the 3-month follow-up (magnet) tracing, the initial pacing artifact, The second and third ventricular complexes are of different mor-
which by definition is the atrial pacing artifact, results in ventricular phology, and stimulation occurs following the second pacing artifact, ie,
stimulation. Since magnet application results in asynchronous pacing, the ventricular pacing artifact. Also note that the AV interval is shorter.
the ventricular pacing artifact follows at the programmed AV delay and This is most likely the result of the magnet having moved, and with
falls into the terminal portion of the QRS interval. reinitiation of the magnet sequence there is abbreviation of the AV in-
Ventricular stimulation by the atrial lead could only occur if the terval for the first 3 cycles. On chest x-ray the atrial lead had dislodged
leads were reversed in the connector block or if the atrial lead had dis- and was positioned in the vicinity of the tricuspid valve.
lodged into a position where it was in contact with ventricular tissue. A fracture of the atrial lead, answer 4, would not result in ventricu-
Because the tracing was normal postimplant and the patient has not had lar capture by the atrial lead. A loose set-screw on the atrial lead, answer
any surgical procedures in the interim, lead reversal can be eliminated 1, could result in intermittent failure to capture or output on the atrial
from the differential diagnosis (option 3). lead, but could not explain the tracing observed.
Furthermore, an additional clue is evident on close inspection of
the lower magnet tracing (Figure 31.2).
Friedman, Rott, Wokhlu, Asirvatham, Hayes 141

31

Figure 31.2  Follow-up magnet tracing. (Repeated)


Case 32
RESYNCHRONIZATION
142 A 73-year-old female has a history of paroxysmal atrial flutter with variable AV
PACEMAKERS, ICDs, AND CARDIAC
,

block and symptomatic bradycardia. At the time she presented for consideration of
pacing, an ambulatory monitor demonstrated a predominant underlying rhythm of
atrial flutter with intermittent normal sinus rhythm. The patient recorded symptoms
of “light-headedness” and “feeling like I could pass out,” at which times tracings
demonstrated atrial flutter with ventricular response rates of 30 to 38 bpm. A
pacemaker was implanted.

The patient now presents to your institution without her pacemaker identification
card or any other information regarding the device or current programming. The
tracing shown in Figure 32.1 was obtained in the device clinic.

Without the benefit of the programmed parameters, and by analyzing only this single
tracing, you need to attempt to determine the underlying rhythm, the programmed
pacing mode, and if the tracing represents normal or abnormal device function.
Friedman, Rott, Wokhlu, Asirvatham, Hayes 143

32

Figure 32.1  Non-magnet tracing.

Q
: Based on your ECG diagnosis, what is your next step?
1. Make ventricular channel more sensitive
2. Increase ventricular output
3. Lengthen ventricular blanking period
4. Do nothing
144 A Case-Based Approach to Pacemakers, ICDs, and Cardiac Resynchronization

32 1. Make ventricular channel more sensitive


The underlying rhythm is clearly atrial fibrillation/flutter. Determin- pacing interval of 100 to 120 ms, and there is no evidence of ventricular
ing the pacing mode in this example is confusing if not assessed care- oversensing. Also, in the second instance when 2 artifacts are seen, the
fully. The first step is to identify the number of pacing artifacts and the second artifact intermittently captures the ventricle.
chamber(s) being stimulated. Is there 1 pacing artifact per timing cycle In this example note that the backup pacing artifact is consistently
stimulating either the atrium or the ventricle, or are there 2 artifacts per smaller than the initial pacing artifact (Figure 32.2). In the earlier gen-
timing cycle, which would conventionally indicate dual-chamber pacing? erations of this feature, the device was designed such that the initial
Also, note the relationship between the pacing artifacts and intrinsic pulse would be of unipolar configuration and the backup pulse would
rhythm. be of bipolar configuration, which likely accounts for this electrocardio-
In this example, there is evidence of paced and intrinsic ventricu- graphic appearance.
lar complexes, as well as pacing artifact, which sometimes fails to cap- This is a VVIR pacemaker with AutoCapture. There is intermit-
ture. In 3 of 6 cycles with pacing artifacts there is a single artifact and tent ventricular undersensing. Whenever the undersensing occurs and
in the remaining 3 cycles, there are 2 artifacts. In the 3 cycles with 2 a ventricular pacing stimulus is subsequently delivered, the ventricle is
artifacts, the first of the 2 artifacts is preceded by an intrinsic ventricular refractory, resulting in functional failure to capture. The AutoCapture
complex within 80 to 240 ms, suggestive of ventricular undersensing. algorithm recognizes failure to capture when the ventricular pacing
In addition, the second artifact always follows the first at an interval of stimulus is delivered, and a high-voltage backup pulse is delivered 80 ms
approximately 80 ms. later. In the tracing, the second undersensed complex occurs at a longer
The initial assessment could certainly be that this is a dual-cham- interval following the intrinsic beat, and the backup pulse results in suc-
ber pacemaker because the 2 artifacts are temporally related. However, cessful capture. In the other 2 cycles with ventricular undersensing, the
why would dual-chamber pacing be evident in only certain cycles, and backup pulse results in functional failure to capture due to proximity to
what does the 80-ms interval between the 2 artifacts suggest? If this were the intrinsic ventricular beat and myocardial refractoriness.
a dual-chamber pacemaker, an AV interval of 80 ms at what appears to Neither lengthening the ventricular blanking period, answer 3, or
be a lower pacing rate would be very short. For ventricular safety pacing, increasing ventricular outputs, answer 2, would correct this abnormality.
the R to stimulus artifact interval is also typically short in order to either If the abnormality was rarely observed, doing nothing, answer 4, might
rescue the patient from atrial crosstalk or ensure harmless delivery dur- not sacrifice patient safety significantly but if a noninvasive program-
ing ventricular depolarization if crosstalk, in fact, is not present. Howev- ming change can correct a pacing abnormality, it should be utilized.
er, 80 ms would be of shorter duration than the usual ventricular safety
Friedman, Rott, Wokhlu, Asirvatham, Hayes 145

32

Figure 32.2  Pacemaker determination.


Case 33
RESYNCHRONIZATION
146
PACEMAKERS, ICDs, AND CARDIAC
,

Figure 33.1  Abnormal surface telemetry.


Friedman, Rott, Wokhlu, Asirvatham, Hayes 147

33
A 76-year-old pacemaker-dependent man underwent CRT-D placement during the
day. You get a call at 2 am because his nurse is concerned about repeated failures
to capture. You interrogate the Medtronic D224TRK CRT-D pacemaker. The device
is programmed to DDDR 60 to 120 bpm. Both left ventricular and right ventricular
lead thresholds and impedances are normal, and provocative arm maneuvers do not
demonstrate system malfunction. With the left ventricular pacing configuration of
LV tip to LV ring, the threshold is 2 V at 0.5 ms. Because it is a new implant, the LV
output was programmed empirically to 5 V at 0.4 ms. Sensed AV delay is 100 ms and
programmed AV delay is 130 ms. Figure 33.1 demonstrates telemetry strips of concern.

What is the most appropriate course of action?


Q
: 1. Lengthen the ventricular blanking period to prevent crosstalk from inhibiting
biventricular pacing
2. Sample different LV lead configurations to reduce anodal capture
3. Place a temporary pacemaker and make plans for LV lead revision in the morning
4. Confirm the timing of automated capture management by looking at the LV lead
threshold trends
148 A Case-Based Approach to Pacemakers, ICDs, and Cardiac Resynchronization

33 4. Confirm the timing of automated


capture management by looking at the the conduction time from LV pace to RV sense events and confirms
LV lead threshold trends that it is longer than the time from an A pace to a conducted RV sensed
This is a classic example of mistaking an automated capture manage- event. Third, with the first test, the programmed LV output is progres-
ment algorithm for failure to capture in a pacemaker-dependent patient. sively decreased after every 3 biventricular pace support cycles until loss
The current generation of chronic implantable devices has algo- of capture is identified. On all consecutive tests, the device remembers
rithms that allow the self-testing of lead threshold. Such automated cap- where the capture threshold was, and starts the threshold test at an in-
ture algorithms are valuable because devices can be programmed to an terval below the previous output that produced capture and increases
adaptive mode, in which they automatically adjust outputs to preserve the output until capture is regained.
battery life and ensure that pacing output occurs within a margin of The main indication that an algorithm is involved is the regular,
safety. Recognition of these types of algorithms, however, is important periodic nature of the loss of capture. An annotated excerpt from the
for avoiding unnecessary reactions to the associated loss of capture that telemetry in Figure 33.2 demonstrates a pattern consistent with the Left
is observed as part of the automated threshold test, particularly in a Ventricular Capture Management algorithm. The pattern is 3 beats of
pacemaker-dependent patient. pacing support followed by a single LV paced beat (identified by its right
Such automatic capture algorithms for right ventricular leads com- bundloid configuration in V1). In this strip, the LV paced beat actually
monly involve assessment of the ventricular capture by delivering a right occurs more than 100 ms early. If the patient was not in atrial fibrilla-
ventricular pacing stimulus and measuring the amplitude of a sensed tion, and atrial sensing or pacing was present, one would expect a very
evoked response. short-paced atrioventricular delay. The rationale for the early delivery
For left ventricular leads, the automated capture approach may dif- of the LV paced beat is to minimize the chance of competitive pacing
fer, depending on the manufacturer. This example illustrates the Left with intrinsic rhythm. When LV loss of capture is identified, the voltage
Ventricular Capture Management algorithm from Medtronic. The al- output between pace support cycles is progressively increased by 0.125
gorithm detects if a left ventricular pacing stimulus at a given threshold V until 3 consecutive beats are captured at the same output. Once the
results in a right ventricular sense within a specific time frame. If the threshold is successfully determined, if programmed to adaptive mode,
right ventricular sense response does not occur within that time frame, the device can change output to reflect the threshold plus the incremen-
the algorithm considers this a failure to capture at that threshold. This tal safety margin.
particular algorithm usually occurs in the middle of the night and be- Automated capture and threshold search algorithms from differ-
gins with testing of LV-RV and AV conduction time. First, the algorithm ent companies vary. While it is unnecessary to memorize the various
confirms rhythm and rate stability. Second, to make sure that right ven- algorithms, it is important to recognize this behavior as normal device
tricular sensing is attributable to a left ventricular stimulus during test- function. Although these tests are usually completed within 20 to 120
ing rather than a conducted atrial event, the algorithm measures both seconds at nighttime, intermittent tachycardia or arrhythmia can result
Friedman, Rott, Wokhlu, Asirvatham, Hayes 149

in repeated or aborted attempts at automated capture, which could re- branch block configuration is not anodal stimulation. It represents sup- 33
sult in a frequent sense of skipped beats. port pacing that is part of a capture management algorithm. Answer 3
Answer 1 is incorrect because the presence of pacing spikes in the is incorrect. There are no data presented to suggested an elevated LV
absence of capture is not failure to output or inhibition of pacing as threshold or risk of inability to capture the ventricle; the noncaptured
might occur during oversensing on V channel due to crosstalk from the beats are part of the automated threshold test. Importantly in the ab-
atrial channel. During crosstalk, either there would be no ventricular sence of LV pacing, there would still be right ventricular pacing if the
output (and hence, no spike), or a safety pacing spike (which occurs 110 RV lead is functional, so that urgent temporary pacemaker placement
ms after the atrial spike). Answer 2 is incorrect because the left bundle would not be needed.

Figure 33.2  Annotated excerpt from telemetry.


Case 34
RESYNCHRONIZATION
150 The surface telemetry strip in Figure 34.1 is obtained the morning after device
PACEMAKERS, ICDs, AND CARDIAC
,

implantation in a patient with left ventricular dysfunction. The device is a dual-


chamber Medtronic pacemaker programmed to rates of 60 to 140 bpm.
Friedman, Rott, Wokhlu, Asirvatham, Hayes 151

34

Figure 34.1  Surface telemetry strip.

Q
: Figure 34.1 is compatible with which of the following?
1. Ventricular oversensing
2. An algorithm to minimize ventricular pacing
3. Dynamic AV delay
4. Safety pacing due to crosstalk
152 A Case-Based Approach to Pacemakers, ICDs, and Cardiac Resynchronization

34 2. An algorithm to minimize ventricular pacing


This is a classic demonstration of an algorithm designed to minimize tricularly sense occur, the device switches to a dual-chamber mode such
ventricular pacing. Such algorithms may be misinterpreted as device as DDDR with a more appropriately programmed AV delay for a certain
malfunction. period of time. Periodic checks for return of atrioventricular conduction
An annotated version of the rhythm strip is shown in Figure 34.2. are performed to allow the switch back to AAIR.
Initially, the device appears to be in AAIR mode. The principle behind algorithms to minimize ventricular pacing is
The first 2 complexes demonstrate atrial pacing followed by ven- concern that unnecessary right ventricular apical pacing promotes dys-
tricular sensing at a very long AR interval of 280 ms. The device allows synchrony and can predispose to left ventricular dysfunction. Other pac-
this long AR interval until there is failure of atrioventricular conduc- ing avoidance algorithms include the AAISafeR algorithm from Sorin
tion, as seen in the third complex in which a dropped ventricular beat Medical, the Search AV hysteresis function in Boston Scientific defi-
occurs. This triggers a ventricular backup pulse after the next atrial beat brillators, or autointrinisc conduction search in St. Jude devices, which
(fourth complex) This occurs with a short, nonphysiologic AV interval prolong the AV interval.
(80 ms). However, the last (fifth) complex demonstrates an atrial paced Answer 1 is incorrect because the failure to V-pace is not due to
beat followed by an intrinsic QRS, after a markedly prolonged AR delay, intermittent ventricular oversensing but to AAIR mode. Answer 3 is
consistent with reversion back to AAIR mode. incorrect because the marked and sudden variation in AV intervals is
This is a demonstration of the Managed Ventricular Pacing (MVP) not in keeping with algorithms for dynamic AV delay, and there is no
algorithm available on Medtronic devices. The algorithm is designed associated increase in heart rate, as would be expected when dynamic
to promote intrinsic atrioventricular conduction and to minimize ven- AV delay is activated. Answer 4 is not the best answer because although
tricular pacing. As seen here, the algorithm paces in AAIR mode with a ventricular backup pulse with a short AV delay suggests ventricular
ventricular “surveillance.” Ventricular pacing with the next pacing cycle safety pacing, this tracing is not consistent with a typical dual-chamber
occurs only if a nonrefractory atrial event is not conducted. It is readily mode. The AV delay is very long and the atrial-based timing allowing for
identified by a short, nonphysiologic AV delay. If multiple failures to ven- a considerable ventricular pause is unusual.
Friedman, Rott, Wokhlu, Asirvatham, Hayes 153

34

Figure 34.2  Annotated rhythm strip.


154
,
PACEMAKERS, ICDs, AND CARDIAC
RESYNCHRONIZATION

Figure 35.1  Patient’s tracing.


Case 35
Friedman, Rott, Wokhlu, Asirvatham, Hayes 155

35

Q
:
The programmed paced AV interval is 240 ms. Where does
the labeled QRS complex (*) in Figure 35.1 occur?
1. PVARP
2. Crosstalk sensing window
3. Atrial blanking period
4. Ventricular blanking period
156 A Case-Based Approach to Pacemakers, ICDs, and Cardiac Resynchronization

35 2. Crosstalk sensing window


The tracing demonstrates probable atrial undersensing leading to an pacing) that is sensed by ventricular channel and resets the VA interval.
intrinsic QRS that falls within the crosstalk sensing window, in the ab- A serious consequence of ventricular oversensing of atrial events is in-
sence of true crosstalk. appropriate ventricular inhibition with the potential for asystole in a
Crosstalk is the unwanted sensing from a channel in one chamber patient who is pacemaker-dependent.
of far-field events arising from the opposite chamber. It is most com- The interesting element of this case is that crosstalk is not the
monly seen as an afterpotential from atrial chamber (usually after atrial cause of ventricular safety pacing in this patient, rather the source is

Figure 35.2  Annotated tracing.


Friedman, Rott, Wokhlu, Asirvatham, Hayes 157

the timing of the QRS that results from its relation to the atrial pacing result in an absolute atrial blanking period preventing sensing of the 35
artifact. An annotated version of the tracing in Figure 35.2 demonstrates afterdepolarization of the pacing stimulus. In some devices, even sensed
2 sinus complexes with intrinsic ventricular conduction. Next, there is atrial events may initiate an atrial blanking period. Answer 4 is incor-
an atrial pacing artifact. Because of baseline noise in the tracing, it is rect; during the ventricular blanking period, nothing can be sensed on
difficult to tell if this pacing artifact is delivered as a result of atrial the ventricular channel.
undersensing or if it is occurring simultaneously with an intrinsic atrial
event. An intrinsic QRS follows, and it occurs during a ventricular pe-
riod known as the crosstalk sensing window (Figure 35.3). The typical
response to sensing during this time period is ventricular safety pacing,
which is readily identified here by the foreshortened atrioventricular in-
terval (100 to 120 ms).
This question assesses your knowledge of event timing on the ven-
tricular channel. On the ventricular channel, a ventricular sensed or
paced event creates a ventricular refractory period. On the ventricular
channel, after an atrial paced event, there is a cross-channel ventricular
blanking period, which is usually followed by a crosstalk safety pacing
window and an alert period. Signals sensed during the crosstalk sensing
window (usually <120 ms from the atrial pacing output) are considered
nonphysiologic due to the close coupling interval and may be caused by
atrial pacing afterpotentials. Additional events sensed in the crosstalk
sensing window may be noise, premature ventricular complexes, or ven-
tricular complexes resulting from undersensed atrial events.
Answer 1, PVARP, is incorrect; this represents the period of time
after a paced or sensed ventricular event during which atrial events are
refractory sensed events and do not affect the timing of events. It is re-
lated to sensing of events on the A channel, rather than the V channel.
Answer 3, atrial blanking period, is incorrect. Again, it reflects a time
window on the atrial channel. On the atrial channel, a paced event may Figure 35.3  The crosstalk sensing window.
Case 36
RESYNCHRONIZATION
158 A patient with a Medtronic single-chamber defibrillator receives a shock, and you
PACEMAKERS, ICDs, AND CARDIAC
,

review the EGM printout (Figure 36.1).


Friedman, Rott, Wokhlu, Asirvatham, Hayes 159

36

Figure 36.1  From top to bottom are shown the ventricular


near-field channel, marker channel, and intervals, recorded
from a patient who was shocked.

Q
:
What is the best explanation for the cause of the shock
delivery after arrhythmia termination?
1. Prolonged rapid atrial fibrillation
2. Lead fracture
3. SVT-VT discrimination algorithm error
4. Oversensing during reconfirmation
160 A Case-Based Approach to Pacemakers, ICDs, and Cardiac Resynchronization

36 4. Oversensing during reconfirmation


This tracing demonstrates T-wave oversensing during reconfirmation. devices forward, reconfirmation is based on the detected tachycardia
Initially, a tachycardia is detected in the VF zone (“FD” in Figure 36.2). cycle length plus 60 ms
Following detection, it spontaneously terminates (asterisk in figure). The When T-wave oversensing occurs during intrinsically conducted
tachycardia is likely ventricular in origin. In the near-field EGM, note rhythms, inappropriate shocks may result due to reconfirmation errors
the Q waves present at EGM onset during tachycardia, but not during (as in the present case) or due to doubling of the heart rate for complexes
normal rhythm (likely sinus rhythm). It is important to bear in mind, for which both QRS and T wave are sensed. When T-wave oversensing
however, that using the near-field EGMs alone, supraventricular tachy- occurs only during pacing, inappropriate shocks do not occur, but the
cardia cannot be reliably distinguished from ventricular in 5% to 10% of pacing rate is slowed as the next paced complex is timed off of the over-
tachycardias. In addition, because 30 to 40 ms of cycle length variability sensed T wave instead of the QRS.
is present, rapid atrial fibrillation cannot be completely excluded. After Answer 1 is incorrect because the patient got shocked despite tachy-
tachycardia reverts to baseline rhythm, the capacitor finishes charging at cardia termination, and because the rhythm is mostly likely ventricular.
“CE” (charge end). Following CE, there is an oversensed T wave (arrow), Answer 2 is incorrect because there are no abnormal high-frequency
which leads to miscalculation of VV interval as 430 ms. In Medtronic potentials to suggest lead fracture as a source of oversensing, and the
ICDs (up to Protecta), reconfirmation requires 3 intervals longer than only oversensed event occurred during the T wave. Answer 3 is incorrect
the slowest tachyarrhythmia zone plus 60 ms. This minimizes the risk as there no evidence of SVT-VT discriminator use or error in this case.
of undertreatment, at the expense of being “trigger happy.” In Protecta
Friedman, Rott, Wokhlu, Asirvatham, Hayes 161

36

Figure 36.2  Annotated tracing. TWOS = T wave oversensing.


Case 37
RESYNCHRONIZATION
162 Figure 37.1 shows a tracing recorded during ICD testing at implantation.
PACEMAKERS, ICDs, AND CARDIAC
,

Figure 37.1  Tracing obtained during implant testing. From top to bottom are shown
surface, marker channels, and near-field ventricular channel.
Friedman, Rott, Wokhlu, Asirvatham, Hayes 163

37

Q
:
Based on this tracing, which of the following is most
accurate?
1. The rhythm is normal and artifact is present
2. Underdetection of ventricular fibrillation is present
3. The defibrillation threshold is elevated
4. The ICD is functioning properly
164 A Case-Based Approach to Pacemakers, ICDs, and Cardiac Resynchronization

37 2. Underdetection of ventricular fibrillation


is present appears to be a one-to-one correlation between surface EGM deflec-
The tracing shows an induced episode of polymorphic ventricular tachy- tions, intracardiac near-field recordings, and sensed events, suggesting
cardia/ventricular fibrillation (VT/VF). As shown in Figure 37.2, the that reprogramming the zones so that this arrhythmia would fall in the
episode begins with induction of polymorphic VT/VF with high-fre- VF rather than fast VT zone would resolve the issue. Following the first
quency burst pacing. Detection occurs in a fast VT zone (undersensing device detection (first blue circle in Figure 37.2), ineffective ATP is de-
of the ventricular arrhythmia, as it should be detected in the VF zone), livered during PMVT (rectangle). The ongoing arrhythmia is redetected
leading to ineffective burst pacing instead of defibrillation shock. Un- and the capacitor charge completed (charge end). Immediately following
dersensing may occur due to combinations of program parameters (rate, charge end, the near-field EGM gets smaller, leading to undersensing of
sensitivity, or duration), low-amplitude EGMs, slew, drug effects, local the arrhythmia, leading the device to classify the episode as terminated
tissue injury, and postshock tissue changes (Swerdlow CD, Friedman (in Figure 37.3, note emergence of pacing, VP). An external rescue shock
PA. Advanced troubleshooting: part II. PACE. 2006;(29):70-96). Several terminates the episode.
of these causes are seen during the course of this tracing. Initially, there
Friedman, Rott, Wokhlu, Asirvatham, Hayes 165

37

Figure 37.2  Tracing obtained during implant testing. From top to bottom are shown surface, marker channels, and near-field ventricular
channel. Blue circles indicate inappropriate detection in fast VT zone instead of VF zone. The rectangle shows inappropriate delivery of
antitachycardia pacing (TP) during polymorphic VT.
166 A Case-Based Approach to Pacemakers, ICDs, and Cardiac Resynchronization

37 Figure 37.3 magnifies the EGMs on the bottom panel of the trac- We have no information about the defibrillation threshold, which
ing. Despite continued easily visible far-field ventricular EGMs, the is the amount of shock energy needed to terminate VF by the device, as a
near-field EGM amplitude decays and is undersensed. This suggests a device shock was not delivered (so answer 3 is incorrect). This is not arti-
different lead position, placing a new electrode, or using a different sens- fact (making answer 1 incorrect), and device function is not functioning
ing configuration (if available) is preferable to programming to increas- properly (answer 4), since VF was not properly detected!
ing sensitivity. Generally, VF is not undersensed when the sinus rhythm
R wave is greater than 5 to 7 mV.
Friedman, Rott, Wokhlu, Asirvatham, Hayes 167

37

Figure 37.3  Magnification of the EGMs in the bottom panels. The arrows indicate the near-field EGM (sharp signal). Note that it is getting
progressively smaller, until it is undersensed (asterisk), although the far-field EGMs are unchanged. This suggests the EGM at the lead
site is not suitable for sensing VF. Continued undersensing results in delivery of a pacing impulse at the +. Despite the fact that a pacing
impulse is delivered, a VS marker is placed. That results since the ventricular paced event at + is actually a safety pace event (note that it is
immediately preceded by an AP event). This indicates that the ventricular EGM actually was sensed, but during the safety pace window that
follows every atrial paced event.
Case 38
RESYNCHRONIZATION
168 The following fluoroscopic images were obtained at the the time of ICD implant.
PACEMAKERS, ICDs, AND CARDIAC
,
Friedman, Rott, Wokhlu, Asirvatham, Hayes 169

38

Figure 38.1  Left anterior oblique (LAO) and right anterior oblique (RAO) images of lead positions.

Q
:
Where is the defibrillator lead (arrow) shown in
Figure 38.1?
1. In the right ventricle
2. Dislodged from the ventricle to the atrium
3. In the coronary sinus
4. In the azygous vein
170 A Case-Based Approach to Pacemakers, ICDs, and Cardiac Resynchronization

38 4. In the azygous vein


The lead in Figure 38.1 indicated by the arrow is in the azygous vein. Be- lead position in Figure 38.1. Ultimately, a coronary sinus lateral branch
cause the apex of the heart points approximately toward the left nipple, (Figure 38.4) rather than the azygous vein or coronary sinus main body
the LAO image separates the right and left cardiac chambers, with the was utilized in this patient to overcome a high defibrillation threshold.
right-sided chambers on the left side of the screen. The lead the arrow Coronary sinus lateral branches are the preferred sites for cardiac resyn-
points to is right behind the heart in this view, in the center of the car- chronization LV pacing leads, but are not commonly used for defibrilla-
diac silhouette. The other lead is in the right ventricle and is coming to- tion coils.
ward us in the image. Note that the LAO view is identified as the sternal
wires on the left side of the image, and spine to the right of the sternum,
consistent with the x-ray tube tilted toward the left nipple.
The RAO view separates atrial structures (left of image) from ven-
tricular structures (right of image). In this case the lead without the ar-
row is seen coming down the superior vena cava and crossing the valve,
moving to the right on the image toward the RV apex. The lead with
the arrow is near the spine, which in the RAO view is to the left of the
sternal wires.
The lead of interest is in the azygous vein, which joins the superior
vena cava posteriorly, just cranial to the right main stem bronchus. It is
cannulated by probing with a wire and a curved catheter such as a mul-
tipurpose or JR-4 (Figure 38.2). Due to its central location posterior to
the heart, in some patients it may offer a desirable defibrillation vector
(posterior/azygous coil to anterior can).
Answer 1 is incorrect. The lead is not in the right atrium. In Figure
38.1 the right atrial lead is clearly seen in the RAO image, curled upward
with a typical J shape. Note that in the LAO panel only part of the atrial
lead is seen. Answer 2 is also incorrect. The RV lead is seen in all of the Figure 38.2  Left: multipurpose sheath (top arrow) has engaged the
azygous vein and a wire advanced into the vein. Following this, a peel-
fluoroscopic images as the lead with a coil that is not indicated by the
away sheath was placed over the wire to permit lead placement in the
arrow. vein. Right: the course of the azygous vein. Cooper JA, Smith TW. How
Answer 3 is incorrect. The position of the main body of the coro- to implant a defibrillation coil in the azygous vein. Heart Rhythm. 2009;
nary sinus is shown in Figure 38.3, which is clearly distinct from the 6(11):1677-1680.
Friedman, Rott, Wokhlu, Asirvatham, Hayes 171

38

Figure 38.3  Defibrillation coil placed in the coronary


sinus (CS, arrow). Left: the left anterior oblique view
shows the lead coursing around the expected location of
the mitral valve. Right: the right anterior oblique view.
The ventricles are to the right of the CS lead, and the
atria are to the left. The RV lead is partially obscured by
the vertical bar. Ordinarily, in the absence of a lead in
the CS, a lucency is seen during fluoroscopy due to AV
groove fat, which can be useful for identification and
cannulation of the CS.

Figure 38.4  Defibrillation coil placed in a lateral branch


of the coronary sinus. This site resulted in an acceptable
defibrillation threshold for this patient. Left: in the
left anterior oblique view, the lead is seen to closely
follow the cardiac silhouette. A shock between the
2 coils shown likely effectively includes all of the LV
myocardium. Right: the lead can be seen progressing
toward the right ventricular apex in the right anterior
oblique view.
Case 39
RESYNCHRONIZATION
172 A 75-year-old male has had a dual-chamber ICD implanted for the past 9 years due
PACEMAKERS, ICDs, AND CARDIAC
,

to dilated cardiomyopathy, syncope, and inducible ventricular tachycardia. A routine


remote monitoring transmission revealed the nonsustained episode shown in Figures
39.1 and 39.2. Just prior to the episode, the patient sent a remote monitoring
transmission revealing multiple episodes of VT/NSVT. The patient’s wife stated that
the he had not been feeling well and was advised to see his local physician.

Tachy parameters:
• VT zone: 140 bpm (430 ms); initial detection: 16 intervals; therapies: ATP x 9,
35-J CV x 3
• FVT zone via VT: 167 bpm (360 ms); therapies: ATP x 6, 5-J CV x 1, 35-J CV x 3
• VF zone: 188 bpm (320 ms); initial detection: 18/24 intervals; therapies: 35-J
shock x 6
• PR logic: AFib/Aflutter and sinus tach: on; other 1:1 SVTs off, SVT limit 290 ms
• Stability: 40 ms; 1:1 VT-ST boundary: 50%; high rate timeout: off

Brady parameters:
• Mode: DDIR
• Pacing rate: 60 to 85 bpm
Friedman, Rott, Wokhlu, Asirvatham, Hayes 173

39

Figure 39.1  Patient’s remote transmission of nonsustained episode interval plot. Legend: V-V: interval (ms) from one
ventricular event to the next. A-A = interval (ms) from one atrial event to the next; FVT = fast ventricular tachycardia zone
interval; TS = interval in VT zone; VF = ventricular fibrillation zone interval; VS = V-V interval below detection zones; VT =
ventricular tachycardia zone interval. If the V-V interval measures 430 ms, the event will not count towards VT detection;
the V-V interval must be <430 ms.
174 A Case-Based Approach to Pacemakers, ICDs, and Cardiac Resynchronization

39

B
Friedman, Rott, Wokhlu, Asirvatham, Hayes 175

39

Figure 39.2 (A–C)  Patient’s remote transmission


of  nonsustained episode electrogram.

Q
:
The episode lasted 20 seconds. Why was therapy
not delivered?
1. The rhythm was not ventricular tachycardia or fibrillation
2. Initial detection was not met due to the stability detection enhancement
3. High rate timeout was programmed off
4. Therapy was not programmed as on in the VT zone
176 A Case-Based Approach to Pacemakers, ICDs, and Cardiac Resynchronization

39 2. Initial detection was not met due to the stability


detection enhancement checked with the stability criterion once the VT event count reaches at
The rhythm is clearly ventricular tachycardia. Note that the ventricular least 3. An interval is declared unstable if the difference between it and
rate is faster than the atrial rate, and more ventricular than atrial EGMs any of the 3 previous intervals is greater than the programmed stability
are present. Due to the stability detection enhancement the VT counter interval, in this case 40 ms. If the device classifies an event as unstable,
was reset (noted at event *0). Thus the event counter did not reach the it is marked as a normal sensed event (“VS” marker), and the VT event
16 consecutive events needed for initial detection. counter resets to zero. Look at the first 3 “TS” events in this episode,
Stability is used to prevent treatment of atrial fibrillation with a with intervals of 420 ms, 390 ms, 380 ms, and 370 ms (Figure 39.2A).
rapid ventricular response that falls in the VT and/or FVT zones, since The difference between the first interval (420 ms) and fourth interval
typically AF has highly variable R-R intervals, whereas VT is regular. (370 ms) is 50 ms, which is greater than the programmed stability of 40
All ventricular intervals in the VT or “FVT via VT” detection zones are ms, so the event count was reset to zero, thus preventing the VT event
Friedman, Rott, Wokhlu, Asirvatham, Hayes 177

counter from reaching the required 16 consecutive events to declare a timeout forces therapy to be delivered after a VT has been detected and 39
VT episode. To treat this VT, the programmed stability value should be sustained for the duration timer. Since initial detection was not met,
increased, or stability turned off. the high rate timeout was never started, so answer 3 is incorrect. In this
Stability is nominally set for 30 msec. However, in the setting of case it would not have timed out and allowed therapy, as it was not pro-
antiarrhythmic drugs, there may be greater variability in R-R intervals grammed on. Therapy was indeed programmed as on in the VT zone,
during VT, leading to undertreatment (as in the present case) with the so answer 4 is incorrect.
use of stability. Looking at the episode plot, it appears the VT hovered right below
Answer 1 is incorrect, since the rhythm is ventricular tachycardia the VT zone lower rate limit for 15 seconds before the EGM recording be-
with A:V dissociation with the atrial median cycle length of 800 ms gan. Besides reprogramming the stability interval, the VT zone detection
and a much faster ventricular median cycle length of 390 ms. High rate rate should be lowered if delivering therapy to this slower VT is desired.
Case 40
RESYNCHRONIZATION
178
PACEMAKERS, ICDs, AND CARDIAC
,

Figure 40.1  Tracing recorded during a tachycardia. From top to bottom are shown
the atrial channel, ventricular channel, and markers.
Friedman, Rott, Wokhlu, Asirvatham, Hayes 179

40

Q
:
In the tracing shown in Figure 40.1, which of the following
is true?
1. Sinus tachycardia is appropriately rejected
2. Sinus tachycardia is inappropriately shocked
3. Ventricular tachycardia is inappropriately rejected
4. Ventricular tachycardia is appropriately shocked
180 A Case-Based Approach to Pacemakers, ICDs, and Cardiac Resynchronization

40 1. Sinus tachycardia is appropriately rejected


The tracing depicts the use of the sudden onset and morphology detec- ing VT does not match the baseline template, and AF morphology does)
tion enhancements to distinguish sinus tachycardia from VT in a St. are available. In the V = A rate branch, as in the present case (SVT= in
Jude ICD. In the top panel of Figure 40.2 the rhythm is sinus tachycar- the tracings, and circled in diagnosis summary, top right box), sudden
dia with occasional premature atrial complexes (vertical arrows). Note onset and morphology are used to distinguish VT from SVT (most com-
that the PACs advance the next ventricular event, consistent with an monly sinus tachycardia) by differences in the onset pattern (VT is sud-
atrially driven rhythm, but that PVCs are also present. The tachycardia den, sinus rhythm is gradual) and in morphology.
is detected in the VT1 zone (box, inset above right). SVT indicates the In Figure 40.2, the onset detection enhancement indicates SVT
tachycardia was sorted to the V=A rate branch classification. Figure 40.2 (gradual onset consistent with sinus rhythm) as does morphology (match
is an annotated version of the episode. scores of 100, with check marks to indicate tachycardia matches baseline
St. Jude ICDs initially sort rhythms by the relative rates in the atria template). Since the logic used in the V = A rate branch is “Any,” if ei-
(A) and ventricles (V). If V > A, VT is declared. Different SVT-VT dis- ther morphology or sudden onset had indicated VT, therapy would have
criminators are used based on rate branch sorting if V = A or V < A, to been delivered. However, since both indicated SVT, therapy is withheld.
enhance the discriminator’s sensitivity and specificity. In the V < A rate The correct answer is therefore number 1. Answer 2 is incorrect, as
branch, stability (in which regular intervals distinguish VT from atrial sinus tachycardia is not shocked. Answers 3 and 4 are incorrect, since
fibrillation, which is irregular) and morphology (ventricular EGM dur- VT is not present.
Friedman, Rott, Wokhlu, Asirvatham, Hayes 181

40

Figure 40.2  Sinus tachycardia appropriately rejected by the ICD. The vertical arrows indicate a PAC that is tracked by the ventricle. The horizontal arrows
are the same length, showing that the ventricular complex marked by the asterisk came earlier than expected, making this a premature ventricular (or rarely,
junctional) complex. The morphology is unchanged with the early ventricular complex (note score of 100, indicating perfect match with baseline template),
but morphology discrimination using the near-field may fail to distinguish SVT and VT in up to 10% of arrhythmias. The third from last complex has a lower
match score (76) and a PR interval shorter than other complexes, suggesting another PVC, possibly fused with the sinus complex.
Case 41
RESYNCHRONIZATION
182
PACEMAKERS, ICDs, AND CARDIAC
,

Figure 41.1  Morphology analysis. Adapted with permission from Swerdlow CD,
Friedman PA. Advanced ICD troubleshooting: part I. PACE. 2005;28:1322-1346.
Friedman, Rott, Wokhlu, Asirvatham, Hayes 183

41
The morphology analysis during device interrogation is shown in Figure 41.1
for a patient with a St. Jude Atlas dual-chamber defibrillator who has a history
of atrial arrhythmias.

Q
:
What is the best explanation for the findings on the strip
in the right panel of Figure 41.1?
1. Alignment error in the setting of intermittent pacing
2. Rate-related bundle branch aberrancy
3. Morphology mismatch by using paced beats as a morphology template
4. Failure to identify template match during ventricular tachycardia
184 A Case-Based Approach to Pacemakers, ICDs, and Cardiac Resynchronization

41 1. Alignment error in the setting of intermittent


pacing to 100 with reportable increments of 1%). The “B” denotes “brady” for a
This is a case of morphology mismatch due to an alignment error in the paced beat. The “S” denotes “sensed” for a sensed beat. Given a similar
setting of intermittent pacing. appearance of these beats to surface ECG and the intracardiac ventricu-
EGM morphology analysis is one means of detection enhance- lar EGMs in sinus rhythm, the 0% match with the sinus rhythm template
ment for discrimination of supraventricular tachycardia from ventricular is unexpected and probably incorrect. This is in contrast to Figure 41.3,
tachycardia. Reviewing the annotated rhythm strips in Figure 41.2 (left which demonstrates an appropriate morphology mismatch when a sensed
panel), the patient is in sinus rhythm. Atrial and ventricular depolariza- premature ventricular complex differed from the normal QRS template.
tions are intrinsic, as indicated by the “P” and “R” shown on marker The failure to match morphology to template may be due to a va-
channels (rather than an “A” or a “V,” which indicate paced events). riety of causes, including:
The checkmarks indicate that the current EGM matches a previously
stored template. To achieve a match, the degree of similarity between the • True morphologic differences:
evaluated EGM and the template must equal or exceed a programmable • Rate-related bundle branch aberrancy
percentage match threshold (nominally 60%). • Ventricular bigeminy
In the right panel of the figure, there is a 0% template match, and • Postshock aberrancy (occurs during redetection)
corresponding “X” to indicate a failure to template match with every • Inaccurate template
other beat. The near-field intracardiac ventricular EGM demonstrates • Incorrect morphology discrimination:
alternating wide and narrow ventricular complexes. This is due to paced • Alignment errors (some component of QRS is
complexes alternating with intrinsic conduction, as demonstrated by the incorrectly assigned leading to a phase shift of all
marker channels that show PV and PR. Interestingly, this alteration is additional points)
not readily seen on the surface EGM, which may be attributable to fu- • Clipping (EGM truncation/clipping due to saturated
sion, or to the fact that only a single lead is shown. amplifiers)
Morphology template analysis does not apply to the paced beats, • Artifact on signal (lead noise or myopotential
hence the absence of a checkmark or X. However, for the narrow beats, interference, may be ameliorated by switch to near-
the device reports no template match (quantified as 0% on a scale of 0 field or far-field EGMs)
185

41

Figure 41.2  Annotated morphology analysis.


186 A Case-Based Approach to Pacemakers, ICDs, and Cardiac Resynchronization

41 Here we show the potential of intermittent pacing to cause an


alignment error. Malalignment is simply the failure of critical elements
of the QRS complex to align with a template. Each company has its
own approach to alignment. In this St. Jude device, alignment depends
on the value of sensing threshold crossing points, as determined by
Automatic Sensitivity Control, in comparison to some preestablished
template (Figure 41.4). When the previous beat is sensed, the sensitivity
threshold for the next beat begins at some percentage of the sensed R-wave
amplitude and decays over time. When the previous beat is paced, the
threshold for the next beat depends on some voltage that is either preset
or algorithmically determined. In this case, there is a small deflection

Figure 41.3  Appropriate failure to match due to PVC.


187

at the beginning of the ventricular EGM. The differences in the “attack ment error and may be preferable. In patients with rate-related bundle 41
rates” of the sensitivity following paced and sensed ventricular events branch block, pacing at faster heart rates (ie, 120 bpm in an AAI mode)
results in detecting the beginning of the QRS using the small predeflec- during template acquisition and disabling the automatic template fea-
tion in one case, and not in the other. This leads to different starting ture permit use of the morphology SVT-VT discriminator.
points and thus alignments for the EGMs following paced complexes. Answer 2 is incorrect because there is no evidence of rate-related
The consequence of 2 different starting points, in this case, is a shift bundle branch aberrancy. There, wide beats are paced. The narrow beats
in the EGM and thus total malalignment (0% match) despite identical are intrinsically conducted. Answer 3 is incorrect because narrow beats
EGMs. from the intrinsic conduction system are the morphology templates. The
Because the alignment error is observed only following paced com- paced beats do not impact the template; they impact the sensing of the
plexes, it has uncertain clinical significance during sensed tachycardia. QRS of interest. Answer 4 is incorrect because there is no evidence of
However, reprogramming sensitivity settings may eliminate the align- ongoing tachycardia.

Figure 41.4  Sensing threshold crossing points.


Case 42
RESYNCHRONIZATION
188 A 77-year-old woman transmits a routine remote monitor transmission that
PACEMAKERS, ICDs, AND CARDIAC
,

demonstrates 4 nonsustained episodes of ventricular fibrillation occuring during


the 4 months since her last transmission. The device implanted is a Guidant CRT-D
connected to Medtronic leads. A new RV defibrillation lead was implanted 8 months
prior to this transmission as a result of failure of the original RV lead. The failed
lead was capped. In-clinic device interrogation just 4 months prior to transmission
revealed no episodes of tachyarrhythmias and normal device function.

Device settings:
Tachy parameters:
• VT zone: 185 bpm; detection duration: 2.5 s; detection enhancements: off
• Therapies: ramp ATP x 1 sequence of 8 pulses at 88%, 41-J shock x 5
• VF zone: 205 bpm; detection duration: 1.0 s
• Therapies: 41-J shock x 8
189

42
Brady parameters:
• Mode: DDDR
• Pacing rate: 50 to 120 bpm
• Ventricular pacing: RV + LV.
• LV pace and sense configuration: LV tip to coil

Device interrogation revealed the EGM and daily measurement trends shown in Figures
42.1, 42.2, and 42.3.
190 A Case-Based Approach to Pacemakers, ICDs, and Cardiac Resynchronization

42

Figure 42.1  Episode EGM.


191

42
Figure 42.2  Daily measurements data table.
192 A Case-Based Approach to Pacemakers, ICDs, and Cardiac Resynchronization

42

Figure 42.3  Daily measurements graph.


Friedman, Rott, Wokhlu, Asirvatham, Hayes 193

42

What is the presumed cause of these nonsustained


episodes and abrupt increases in RV and LV pace

Q
:
impedances in late May 2010? (Of note, pacing thresholds
unchanged from January to September 2010.)
1. RV lead failure
2. Loose set screw
3. Exposure to electromagnetic interference
4. Excessive fibrosis
194 A Case-Based Approach to Pacemakers, ICDs, and Cardiac Resynchronization

42 1. RV lead failure
The RV defibrillation lead had failed. As a result of the LV lead utiliz- noise deflections are seen briefly on the far-field signal—suspicious for
ing the RV lead to both pace and sense, impedance increases were seen a process affecting several conductors, eg, lead damage or contact with
on both the RV and LV pacing leads. Pocket manipulation also repro- an abandoned lead. This series of observations raises the possibility that
duced noise on the shock (far-field) EGM. There was question as to why the patient was moving her arm (leading to myopotentials on the far-
the shock EGM also showed noise, especially since the shock imped- field EGM, which includes the pulse generator as electrode), and that the
ance had not changed. The low-amplitude noise on the far-field (shock) mechanical motion subsequently triggered the make-break potentials on
EGM that begins a few cycles before the noise on the near-field (RV the near-field and far-field EGMs.
pace) EGM most likely reflects myopotentials. In addition, some large
195

The RV lead was removed and a new lead was implanted. The consistent values, a make-break connection was not thought to be the 42
failure was believed to be in the RV ring electrode, since both the RV problem, ie, answer 2 is incorrect. Whereas exposure to electromagnetic
and LV pacing utilized this electrode, and the RV pace (near-field) EGM interference (answer 3) may result in nonsustained episodes, it would
consistently demonstrated noise. not cause the impedance changes that were then maintained. Answer 4
Chest x-ray demonstrated that all pins were fully inserted into the is incorrect, for excessive fibrosis at the lead or myocardial interface is
header of the pulse generator, and verified with fluoroscopy prior to the the cause of exit block, which presents as higher pacing thresholds, not
lead revision. As the impedances rose suddenly but then plateaued at a change in impedance.
Case 43
RESYNCHRONIZATION
196 A 68-year-old male received dual-chamber pacemaker sinus node dysfunction
PACEMAKERS, ICDs, AND CARDIAC
,

with syncope.

The transtelephonic pacemaker tracing shown in Figure 43.1 was obtained 3 weeks
postimplant.

Device settings:
• Mode: AAIRóDDDR
• Pacing rate: 60 to 130 bpm
• Paced AV delay: 200 ms
• Sensed AV delay: 170 ms
Friedman, Rott, Wokhlu, Asirvatham, Hayes 197

43
Figure 43.1  Panel A:
Nonmagnet TTM rhythm
strip.

Figure 43.1  Panel B:


Nonmagnet TTM rhythm
strip.

Q
:
Why are there variations in the paced and sensed
AV delays?
1. AV search hysteresis
2. Magnet application
3. Intermittent ventricular failure to capture
4. Ventricular undersensing while in Managed Ventricular Pacing (MVP) mode
198 A Case-Based Approach to Pacemakers, ICDs, and Cardiac Resynchronization

43 4. Ventricular undersensing while in Managed


Ventricular Pacing (MVP) mode will search for intrinsic conduction. If there is none, the device remains
The programmed mode is indeed MVP, in which the device switches be- in DDDR mode for 2 minutes, rechecking again. This exponentially
tween AAIR to DDDR depending on the presence of intrinsic R waves increases until the interval reaches 16 hours, at which time the device
(Figure 43.3). In this mode, when the device is in AAIR and believes will continue to check every 16 hours until the mode is reprogrammed.
an atrial paced event is not followed by a ventricular sensed event, the Note that the paced QRS complexes in events 2, 6, and 8 with the
device allows for this loss of conduction to occur once, and then follows shorter-paced AV delay of 80 ms result in a captured ventricular paced
with an atrial paced/ventricular paced event with a shortened paced complex, while the longer-paced AV delay of 200 ms in events 9 to 13
AV delay of 80 ms (Figure 43.3) The device then stays in AAIR mode results in a slightly fused ventricular paced complex (Figure 43.2). This
and allows for an atrial paced event again with no ventricular intrinsic is because the shortened AV delay fully captures the ventricle, but the
conduction. If 2 of 4 events occur with no ventricular conduction, the longer AV delay allows intrinsic AV conduction resulting in fusion of the
device switches to DDDR mode. After 1 minute of DDDR, the device paced and sensed complexes.
199

43

Figure 43.2  Nonmagnet TTM rhythm strip.


200 A Case-Based Approach to Pacemakers, ICDs, and Cardiac Resynchronization

43 Answer 1 is incorrect, as the device goes from immediately sensing ufacturer. However, answer 2 is incorrect for this Medtronic EnRythm
the intrinsic QRS in events 1, 3 to 5, and 7 to a shortened AV delay in P1501DR device would respond to magnet application by AV pacing
events 2, 6, and 8. Then, once the consistent pattern of AV-sequential at 85 bpm with the programmed AV delay of 200 ms, and this rhythm
pacing with the programmed paced AV delay of 200 ms begins in events strips shows AV pacing at the sensor-driven rate of 68 bpm.
9 to 13, the AV delay remains at the programmed AV delay and does not Answer 3 is incorrect, as each event shows either atrial pacing with
extend out to search for the intrinsic QRS complex to promote ventricu- intrinsic conduction of the QRS complex, or AV pacing with ventricular
lar sensing. capture or fusion of the ventricular pacing output and intrinsic QRS
Transtelephonic rhythm strips usually do show varying AV delays complex.
due to magnet application, dependent upon the specific model and man-
201

43

Figure 43.3  Panel A: MVP mode: Ventricular pacing only as needed in the presence of transient loss of
conduction. Reproduced with permission of Medtronic, Inc. Panel B: Continued MVP mode: Ventricular
support if loss of AV conduction is persistent. Reproduced with permission of Medtronic, Inc.
Case 44
RESYNCHRONIZATION
202 A 53-year-old man with a history of atrial fibrillation and a favorable response to
PACEMAKERS, ICDs, AND CARDIAC
,

CRT-D placement recently underwent mitral and tricuspid valve replacements. He


presents for follow-up device interrogation due to worsening heart failure, despite
normally functioning prostheses. The device is programmed to biventricularly pace
VVIR 70 to 130 bpm with an LV offset of –20 ms. Upon interrogation, the LV lead
threshold is 0.5 ms at 2.8 V in the LV tip to RV coil configuration, but there is no
capture in other configurations. Counters report 100% biventricular pacing. Twelve-
lead ECGs are obtained during RV pacing and LV pacing and compared to a presurgical
ECG (Figure 44.1). The corresponding chest x-ray is shown in Figure 44.2.
203

44

Figure 44.1  Electrocardiograms before and after surgery.


204 A Case-Based Approach to Pacemakers, ICDs, and Cardiac Resynchronization

44

Figure 44.2  Chest x-ray after surgery


Friedman, Rott, Wokhlu, Asirvatham, Hayes 205

44

Q
:
What is the likely reason for a new failure to respond
to cardiac resynchronization therapy?
1. Poor LV lead site selection at the time of original implant
2. Insufficient LV-RV offset to account for postsurgical slow exit from the LV pacing site
3. The RV lead is dislodged into the coronary sinus
4. RV anodal capture is occurring due to LV lead dislodgment
206 A Case-Based Approach to Pacemakers, ICDs, and Cardiac Resynchronization

44 4. RV anodal capture is occurring due to LV lead


dislodgment sternum giving rise to a negative deflection in V1 (LBBB
This is a case of postsurgical LV lead dislodgment into the right ventricle pattern). A large positive deflection is seen in lead I and a
with only anodal capture at the right ventricle during attempted LV pac- large negative deflection is seen in lead III.
ing. Thus, resynchronization is absent due to lead dislodgment. • In LV pacing, there is a negative deflection in lead I, and
Although the histogram counters report that biventricular pacing V1 is positive consistent with a RBBB configuration. Lead
is occurring 100% of the time, the 12-lead ECGs during RV and LV III can be variable; it is generally positive with high lateral
pacing demonstrate a problem with the LV pacing vector. The LV pacing positions, and negative with lower/inferior positions.
configuration is nearly identical to the RV pacing configuration. Both • In biventricular pacing, a hybrid vector is expected. The
configurations are of LBBB morphology. In contrast, the presurgical QRS is ideally somewhat negative in lead I, depending on
QRS configuration is of RBBB morphology (positive in lead V1) and lead placement, and may have a right bundle or hybrid
negative in lead I and therefore, is more consistent with biventricular morphology.
pacing.
This case underscores the value of accurately placing critical leads In this case, RV capture occurred when LV pacing was performed.
such as I, V1, or III or using a 12-lead ECG when LV capture is in ques- Pacing always requires closure of an electrical circuit, which includes a
tion during device interrogation. A general understanding of the elec- cathode (the desired pacing electrode) and an anode (the return current
trocardiographic configurations one expects is important too. General electrode). Anodal stimulation or capture occurs when the myocardium
rules include: is stimulated and captured first at the return current electrode. If pacing
is programmed to occur between an LV electrode (cathode) and RV coil
• As a general principle, QRS deflection is negative when (anode) and there is no LV capture, capture occurs only at the RV coil
the wavefront travels away from a specific lead (eg, negative (anode). Thus, in anodal capture, the ECG reflects RV pacing (as in this
QRS in lead I indicates pacing from the lateral LV with case). If RV capture without LV capture is mistaken for LV capture, pac-
the wavefront moving away from the left). A positive QRS ing without resynchronization occurs.
reflects activation toward a given lead (so that RV pacing is Anodal stimulation is favored when the anodal electrode is small
generally positive in lead I). relative to the cathode. Thus the risk is greater with biventricular pace-
• During RV apical pacing, a typical LBBB configuration is makers, in which the anode is a ring electrode, as opposed to with de-
seen. The right ventricle is right behind the sternum. When fibrillators, in which the anode is a large defibrillation coil. The small
it is activated first, the wavefront travels away from the surface area of the ring electrode in pacemakers results in a high current
207

density, increasing the likelihood of capturing the surrounding RV tis- suggests a very apical position. This is confirmed by the chest x-ray. The 44
sue. In addition, anodal stimulation may be more prominent if LV pac- postrevision chest x-ray portable PA view is shown in Figure 44.4. The
ing occurs significantly later than a paced or sensed RV event. LV lead is now lateral and leftward.
Note that if an LV tip-to-ring configuration is used, both anode Answer 1 is incorrect because preoperatively the pacing vector was
and cathode are LV leads so that anodal stimulation is not clinically consistent with LV capture (see Figure 44.1) and the patient responded
important. When pacing in an LV-to-RV configuration, if both cathodal to therapy. Answer 2 is incorrect because although poor offset can in-
and anodal stimulation occur, it is generally of no clinical consequence. crease the RV pacing component of the vector, there was no previous
The main concern is when anodal stimulation occurs alone (without mention of LV exit block, and failure of LV pacing was due to frank lead
capture at the cathode, or LV site). Recognition of isolated anodal stimu- dislodgement into the RV.
lation often requires a 12-lead ECG to assess pacing morphology. Answer 3 is incorrect because the RV lead is in place, as confirmed
In this case, the LV pacing lead was dislodged into the right ven- by the x-ray and the RV paced QRS morphology. The LV lead is dis-
tricle. It is possible that there was direct capture of the RV tissue from lodged into the right ventricle.
the LV lead tip. However, a compelling argument for anodal capture is
that despite significant separation of the RV and displaced LV lead tips
in the right ventricle (as seen in the chest x-ray), the 12-lead pacing con-
figuration is identical in RV and LV pacing. Also, the observation that
the capture only occurred in one configuration (LV tip to RV coil) is
consistent with anodal capture, as direct capture from the LV lead (the
cathode) was never demonstrated.
This case is a good illustration of the importance of careful post-
surgical evaluation of device function, particularly the LV lead.
Postrevision RV pacing, LV pacing, and biventricular pacing vec-
tors are now seen in Figure 44.3. Note that lead I is now negative with
LV pacing and has a hybrid appearance with biventricular pacing. Also
note that LV pacing postrevision results in an RBBB pattern (very posi-
tive QRS in V1) as opposed to the biphasic pattern with RV pacing. The
fact that RV pacing is biphasic in V1 (as opposed to negative/LBBB)
208 A Case-Based Approach to Pacemakers, ICDs, and Cardiac Resynchronization

44

Figure 44.3  Postrevision biventricular pacing, RV pacing, and LV pacing vectors.


209

44

Figure 44.4  Postrevision chest x-ray.


Case 45
RESYNCHRONIZATION
210 An 85-year-old male returns to the device clinic for routine interrogation of his
PACEMAKERS, ICDs, AND CARDIAC
,

dual-chamber pacemaker 3 months after pulse generator change. His pacing leads
have been in place for 9 years. The initial indication for pacemaker implant was
postoperative high-grade AV block with syncope.

Device settings:
• Mode: DDDR
• Pacing rate: 60 to 130 bpm
• Mode switch: on
• Atrial lead: output 2.5 V at 0.3 ms; sensitivity: 0.1 mV;
AutoSense: off
• Ventricular lead: output 1.0 V at 0.4 ms (AutoCapture: on);
sensitivity: 2.5 mV; AutoSense: off
211

45
At the time of the device clinic visit the patient was in atrial fibrillation and current
interrogation reveals a P wave of 0.7 mV and impedance of 390 ohms. Prior atrial lead
measurements were:
• Initial implant (9 years prior): P wave 2.1 mV in sinus rhythm, 529 ohms
• Pregenerator change via device: P wave 0.5 mV, impedance 426 ohms
• During generator change lead testing via analyzer: P wave 0.3 mV, impedance
469 ohms
• Immediately postgenerator change via device: P wave 0.4 mV, impedance
350 ohms

Interrogation reveals 255 episodes of atrial noise reversions, as outlined in


Figure 45.1.
212 A Case-Based Approach to Pacemakers, ICDs, and Cardiac Resynchronization

45

Figure 45.1  Patient’s EGM.


Friedman, Rott, Wokhlu, Asirvatham, Hayes 213

45

Q
:
What is the best explanation for these 255 atrial noise
reversion episodes?
1. Atrial lead sensing failure after pack change
2. Atrial lead pin not fully inserted into header of new device
3. Frequent exposure to electromagnetic interference
4. Oversaturation of the atrial sensing amplifier
214 A Case-Based Approach to Pacemakers, ICDs, and Cardiac Resynchronization

45 4. Oversaturation of the atrial sensing amplifier


This case illustrates one cause for a spurious noise reversion episode. change. Upon interrogation, the sensitivity level was set unusually low at
The tracing shows that the patient is in atrial fibrillation and the device 0.1 mV (typical settings ≥ 0.5 mV). In the setting of frequent atrial events
has appropriately mode switched (AMS) from DDDR to DDIR. Both during atrial fibrillation, the sensing amplifiers were oversaturated, re-
leads appear to sense and pace appropriately. For each cardiac cycle, sulting in a misclassification as noise. Since P waves measured 0.7 to 1.0
there is an initial atrial sensed event (AS) after the postventricular atrial mV during testing, the atrial sensitivity level was reprogrammed to 0.3
refractory period. Subsequent atrial events are sensed but are designated mV. At this level, atrial fibrillation was still detectible—still allowing for
as atrial refractory events (AR/R). Then an atrial noise reversion epi- appropriate mode switching to occur.
sode occurs (A-<-A-Noise). This is inappropriate because is there is no Noise reversion is an algorithm that relies on the fact that rapid
noise or apparent change on the atrial EGM. frequencies are nonphysiologic. In St. Jude Medical devices, signals of
In this case, the most likely explanation for the frequent atrial 50 Hz or faster are classified as noise. In the atrium, as we see here,
noise reversion episodes is a change in sensitivity settings after pack there is typically no automatic programming change; however, sensed
215

events in the atrium are ignored during the episode. In the ventricle, into the header would most likely result in a variable impedance. Inter- 45
these algorithms are protective against oversensing of noise and inap- rogation revealed stable lead measurements and trending since the pulse
propriate pauses. Once a ventricular noise reversion event is triggered, generator change. Answer 3 is incorrect because frequent exposure to
there is “reversion” to an asynchronous ventricular pacing mode, such EMI would most likely result in noise on both leads and only atrial noise
as DOO or VOO. If noise reversion occurs in an ICD, ventricular noise reversion is occurring and stored EGMs fail to demonstrate any noise on
reversion would disable the charge for VT/VF therapy, and detection the ventricular EGM.
enhancements during atrial noise reversion are classified as V>A so VT/
VF therapy would not be withheld.
Answer 1 is incorrect because the atrial sensing threshold has actu-
ally improved slightly since pack change, from 0.3 to 0.4 mV to 0.7 mV.
Answer 2 is incorrect, because a lead pin that was not fully inserted
Appendix 217

Spoiler alert! This appendix identifies the cases in this book by diagnosis, which may suggest or reveal the answers to questions
in the cases. Because we want to encourage readers to approach the cases as unknowns, we are presenting this list as an
appendix rather than as a table of contents and we recommend that you use the appendix only after working through the cases.

1 Diathermy, artifact, oversensing, electromagnetic interference, 2 14 Diagnosis and management of intermittent failure to capture, 56
2 Pacing artifacts, temporary pacemakers, device-device 15 Atrial lead dislodgement, failure to sense, failure to capture,
interaction, 6 programming ATP in single-chamber (atrial only) device, 60
3 Magnet response, magnet mode, 10 16 Inappropriate shocks, lead dislodgement, 64
4 Artifact on surface ECG, 14 17 P-wave oversensing, V lead dislodgement, 70
5 Threshold test and digital surface ECG missing spikes, 18 18 Mode switch surface ECG, pacemaker wide complex
6 Inadvertent LV pacing via patent foramen ovale, normal and tachycardia analysis, 74
abnormal ECG and chest x-ray following pacemaker implant, 22 19 Lead reversal in header,
7 Far-field oversensing, P-wave oversensing, 26 A-V reversal, 78
8 Far-field oversensing, R-wave oversensing, inappropriate 20 Hysteresis, ventricular undersensing, differential diagnosis
mode switch, 30 of pacing at rates below the lower rate limit, 82
9 Pacemaker mediated tachycardia, 34 21 Analysis of diagnostics in CRT device, 92
10 Approach to high rate episode, lead noise, EMI, 38 22 Analysis of diagnostics in CRT device, 98
11 Pacemaker Wenckebach, upper rate behavior, 42 23 Reconfirmation, committed vs. noncommitted shock
12 Surface ECG recognition of fusion, pseudofusion, delivery, 104
and capture, 46 24 SVT-VT discrimination enhancements, shock for sinus
13 Pacemaker timing cycles and blanking periods, distinguishing tachycardia, sustained rate duration, inappropriate shock, 108
pseudofusion from pseudo-pseudofusion, 50 25 Complex transtelephonic monitoring tracing analysis, 114
218 Appendix

26 Ventricular failure to output, 118 36 T-wave oversensing, reconfirmation, inappropriate shock, 158
27 Lead reversal, 122 37 Undersensing of VF by an ICD, 162
28 Far-field R-wave oversensing, AAI pacing mode, 126 38 Management of high defibrillation threshold, recognition
29 Rate-Drop Response, assessment of pacemaker pacing faster of alterative defibrillation lead positions (including coronary
than expected, 130 sinus and azygous vein) by chest x-ray, 168
30 Upper rate behavior, programming to treat chronotropic 39 SVT-VT discriminators (stability), VT underdetection, 172
insufficiency, 134 40 SVT-VT discriminators (onset and morphology), 178
31 Atrial lead dislodgment, 138 41 SVT-VT descriminators (morphology), 182
32 AutoCapture, unexpected pacing artifacts on surface ECG, 42 Differential diagnosis of lead noise, lead failure, 188
algorithmic pacing, 142 43 Managed ventricular pacing, algorithmic pacing, 196
33 LV capture management, algorithmic pacing, 146 44 ECG and chest x-ray analysis of LV lead position, anodal
34 Managed ventricular pacing, algorithmic pacing, 150 stimulation, 202
35 Ventricular safety pacing, atrial undersensing, crosstalk window, 45 Noise reversion, atrial sensing oversaturation, 210
blanking or refractory intervals, pacemaker timing cycles, 154
Index 219

AAIR pacing mode, 60–63, 126–28, automated capture management algorithms, cardiomyopathy
150–53, 198–99 146–49 dilated, 26, 98, 172
AAISafeR algorithm, 152 Automatic Sensitivity Control, 186 ischemic, 72–73, 92–96
AF (atrial fibrillation), 94, 210–15 automatic switched mode (AMS), 32 chest x-ray
algorithm minimizing ventricular pacing, AV block atrial lead dislodgment, 62
150–53 intermittent, 138–41 determining lead position, 24–25, 170–71
amiodarone, 94 pacemaker for, 10–13, 22–25, 34–37 loose connector pin, 121
anodal capture, 202–9 postoperative, 42 postoperative, 204, 207, 209
antitachycardia pacing (ATP), 4, 104–7 AV interval hysteresis, 86–87 connector pins, incomplete insertion,
AT (atrial tachycardia), 94 AV node conduction, amiodarone and, 94 122–23
atrial arrhythmias, 60–63 AV synchrony, 37 crosstalk, 154–57, 156
atrial contraction, premature, 36 axillary vein, 58 CRT. See cardiac resynchronization therapy
atrial far-field events, 126–28 azygous vein, 168–71 (CRT)
atrial fibrillation (AF), 94, 210–15
atrial flutter, 16, 142–45 bundle branch block, right, 22–25 DDD pacing mode, 2–5, 42–45, 112–13,
atrial lead 118–23, 130–33, 134–37
dislodgment, 62–63, 138–41 cardiac resynchronization therapy (CRT) DDDR pacing mode, 14–17, 26–29, 30–33,
far-field R-wave oversensing, 30–33 automated capture management 34–37, 50–55, 122–25, 146–49,
reversal, 80–81, 124–25 algorithms, 146–49 188–95, 198–99, 214–15
atrial refractory period, 36–37, 214 fluid volume measurements, 98–103 DDIR pacing mode, 172–77
atrial sensing amplifier, 210–15 LV lead dislodgment and RV anodal diathermy, 4
atrial tachycardia (AT), 94 capture, 202–9 dilated cardiomyopathy, 26, 98, 172
atrioventricular terms. See AV terms right ventricular lead failure, 188–95
autointrinsic conduction search, 152 VF resumption, 104–7
220 Index

EGM. See electrogram (EGM) heart block, complete, 56–59, 134–37 loss of programmer communication, 12
electrocardiogram (ECG) LVAD (left ventricular assist device), 102–3
algorithm minimizing ventricular implantable cardioverter-defibrillator (ICD)
pacing, 150–53 appropriate rejection of sinus magnet response, 12
anodal capture, 202–9 tachycardia, 178–81 minimized ventricular pacing (MVP)
atrial flutter, 142–45 AT/AF episodes, 92–94 avoidance algorithms
QRS complex in crosstalk sensing DDD pacing mode, 108–13 AAIR to DDDR mode switching, 198–99
window, 154–57 defibrillator lead in azygous vein, 168–71 automated capture management
right bundle branch block, 22–25 electromagnetic interference, 2–5 timing, 152–53
sinus tachycardia vs. VT, 178–81 increase in thoracic fluid volume, 98–103 ventricular undersensing, 196–201
ventricular failure to output, 118–23 lead dislodgment, 72–73 morphology algorithms, EGM, 182–87
electrogram (EGM) lead revision, 68–69
algorithms minimizing ventricular morphology algorithm error, 182–87 noise reversion algorithm, 214–15
pacing, 150–53 nonsustained episode, 172–77
electromagnetic interference, 2–5 oversensing during reconfirmation, OptiVol fluid index, 99, 102
lead reversal diagnosis, 80–81, 124–25 158–61 oversensing. See also T-wave oversensing
morphology analysis, 182–87 postoperative reprogramming failure, lead integrity loss, 4
near-field, 167 95–96 P-wave, 28
normal pacemaker function, 20–21 sustained rate duration timed out, 112–13 R-wave, far-field, 30–33
oversensing during reconfirmation, T-wave oversensing, 66–67, 158–61 ventricular, atrial events, 156–57
158–61 ventricular sensed markers, 26–29
stored, correlation with patient VF underdetection, 162–67 PAC (premature atrial contraction), 36
activities, 40–41 intrinsic beat, surface ECG, 47–48 paced beat, 47– 48
T-wave oversensing, 67, 158–61 ischemic cardiomyopathy, 72–73, 92–96 pacemaker syndrome, 130–33
underdetection of ventricular isometrics, noise artifacts, 132 pacemaker-dependent patients
fibrillation, 162–67 avoidance of welding, 40
ventricular failure to output, 118–23 leads discrepancies in transtelephonic
electromagnetic interference (EMI), 2–5, 40 dislodgment, 202–9 transmissions, 114–17
exertion, 136–37 failure, 4, 188–95 lightheadedness in, 56–59
placement, 24–25, 58 management after implant, 8
failure to capture, atrial, 62 reversal, 80–81, 124–25 pacemaker-mediated tachycardia (PMT),
far-field events, 126–28, 156 left ventricular assist device (LVAD), 102–3 36–37
far-field R wave (FFRW) oversensing, 30–33 Left Ventricular Capture Management, pacemakers
far-field sensing, atrial, 126–28 148, 149 biventricular, 206
Fontan procedure, 60–63 lightheadedness, 56–59, 130–33 connector pin not inserted in header,
fusion beat, 47–50, 76 118–23
Index 221

DDDR pacing mode, 10–13, 14–17, postventricular atrial refractory period stability detection enhancement,
30–33, 50–55, 78–81 (PVARP), 36–37 ICD, 172–77
dual-chamber premature atrial contraction (PAC), 36 Sudden Brady Response (SBR)
AAIR pacing mode, 60–63 premature ventricular contraction algorithm, 132
algorithms minimizing ventricular (PVC), 180 supraventricular tachycardia (SVT), 180
pacing, 150–53 programming head, loss of communication sustained rate duration, 112–13
atrial lead dislodgment, 138–41 with device, 12
atrial sensing amplifier oversaturation, pseudofusion beat, 47, 48, 49, 76 tachycardia. See also ventricular
214–15 pseudo-pseudofusion beat, 49, 54–55 tachycardia (VT)
far-field R-wave oversensing, 30–33 pseudo-Wenckebach behavior, 136–37 antitachycardia pacing (ATP), 4, 104–7
magnet response, 12 P-synchronous pacing, 138 atrial, 94
mode switch occurrences, 30–33 pulmonary edema, 102–3 pacemaker-mediated, 36–37
normal function, 74–77 pulse generator, 122–23 sinus vs. ventricular, 178–81
surface ECG vs. marker channel/ PVARP (postventricular atrial refractory supraventricular, 180
ventricular EGM, 14–17 period), 36–37 telemetry, loss of communication
normal device function, 18–21 PVC (premature ventricular with device, 12
permanent, preoperative evaluation, contraction), 180 thoracic impedance, 102–3
38–41 trace markers, far-field events, 128
single-chamber, 6–9 railroad tracks (tramtracking), 67 tramtracking (railroad tracks), 67
stored EGM correlation with patient rate duration, sustained, 112–13 transtelephonic monitoring, 42–45, 114–17
activities, 40–41 rate smoothing, sensor-driven, 136 transvenous pacing, 24
temporary, undersensing permanent Rate-Drop Response (RDR) algorithm, T-wave oversensing
pacing, 8 130–33 programming to avoid, 67, 68
transtelephonic monitoring, 42–45 R-wave, far-field oversensing, 30–33 during reconfirmation, 158–61
ventricular failure to output, 118–23 single-chamber ICD, 66–67
VVIR pacing mode, 6–9 SBR (Sudden Brady Response) ventricular lead dislodgment, 72–73
VVIR pacing with AutoCapture, 142–45 algorithm, 132 ventricular lead revision, 68–69
pacing, transvenous, 24 Search AV hysteresis, 152 ventricular sensitivity threshold, 82–91
Parkinson’s disease and atrial flutter, 16 sick sinus syndrome with syncope, 14–17
patients. See pacemaker-dependent patients sinus node dysfunction undersensing
PMT (pacemaker-mediated dual-chamber pacemaker, 74–77, 122–25 atrial, 62
tachycardia), 36–37 with syncope, 30–33 temporary pacemakers, 8
post-cardiac surgery ventricular undersensing in MVP ventricular, 90–91, 196–201
AT/AF episodes, 94 mode, 196–201 ventricular fibrillation, 162–67
device management, 8 sinus rhythm, 184–85 upper rate limit, programmed, 44
LV lead dislodgment, 202–9 sinus tachycardia, vs. VT ., 178–81
rate response reprogramming, 96
222 Index

ventricular blanking period, 52, 157 P wave oversensing, 28 sustained rate duration timed out,
ventricular complexes, surface ECG, 46–49 reversal, 80–81, 124–25 112–13
ventricular contraction, premature, 180 T-wave oversensing, 68–69 lead reversal, 80–81
ventricular failure to output, 118–23 ventricular pacing, 36, 150–53 sinus tachycardia vs., 178–81
ventricular fibrillation (VF) Ventricular Rate Regulation (VRR), ventricular threshold test, 20
nonsustained episodes, 188–95 73, 137–40 VF. See ventricular fibrillation (VF)
resumption prior to ATP confirmation, ventricular refractory period, 157 VT. See ventricular tachycardia (VT)
104–7 ventricular safety pacing, 156–57 VVI pacing mode, 82–91, 86–87, 90–91
undersensing by ICD, 162–67 ventricular sensed markers, 26–29 VVIR pacing mode, 6–9, 104–7,
ventricular lead ventricular tachycardia (VT) 114–17, 142–45
alternate venous routes, 58 implantable cardioverter-defibrillator
automatic capture algorithms, 148–49 dual-chamber, 2, 92–96 welding and EMI, 40
dislodgment, 72–73 stability detection enhancement, Wenckebach phenomenon, 44–45
failure, 188–95 176–77

You might also like